Этого треда уже нет.
Это копия, сохраненная 18 января 2022 года.

Скачать тред: только с превью, с превью и прикрепленными файлами.
Второй вариант может долго скачиваться. Файлы будут только в живых или недавно утонувших тредах. Подробнее

Если вам полезен архив М.Двача, пожертвуйте на оплату сервера.
Тред тупых вопросов №152 662981 В конец треда | Веб
Тред вопросов о жизни, Вселенной и всём таком.

Спрашиваем то, за что в других местах выдают путёвку в биореактор. Здесь анонимные учёные мирового уровня критически рассмотрят любые гениальные идеи и нарисованные в Paint схемы.

Предыдущий тут: >>658473 (OP)
https://2ch.pm/spc/res/658473.html (М)

Q: Можно быстрее?
A: Можно упасть в пузырь альбукерке, наса уже почти надула его.

Q: Я начитался охуительных историй про уфологию, че делать, нам жопа?
A: Да, тебе жопа, можешь сгонять в зогач или куда оттуда пошлют.

Q: Что будет с человеком в вакууме без скафандра / если он упадет на черную дыру / попробует ступить на поверхность газового гиганта/солнца?
A: Он умрёт.

Q: Почему бы не привязать ракету к воздушному шару или стартовать с горы?
A: Космос - это не как высоко, а как быстро, большая часть энергии ракеты уходит на разгон вбок.
Подробнее тут https://what-if.xkcd.com/58/ (английский) https://chtoes.li/orbital-speed/ (перевод)
2 662985
>>62981 (OP)
Когда уже изобретут антигравы? Есть подвижки в этой области?
3 662986
>>62985
Для начала нужно понять что такое гравитация.
4 662987
>>62986
А то что известно сейчас чем не устраивает?
5 662988
>>62985
Еще не скоро, если вообще изобретут. По причине, названной вот этим товарищем >>62986
У нас от наблюдения звёзд до полёта в космос сколько времени прошло? 2 тысячелетия? То то и оно
6 662989
>>62988
А как же ускорение прогресса? Технологическая сингулярность? То что раньше занимало два тысячелетия, сегодня занимает джва часа?
7 662990
>>62989
Но звёзды мы как наблюдали, так и наблюдаем. И даже не приблизились пока ни к одной, только "щупаем". Ты понимаешь какое дело:

>ускорение прогресса?


Вся эта катавасия под названием прогресс, имеет свойство замедляться. Я тебе больше скажу, мы в регресс можем уйти вот легко, лет так на 500 или на тысячу. Не хотелось бы разводить политоту, но давай просто возьмём 2 периода:

>1.70е СССР


>2.21й год РФ.


В первом случае было что? На марсе яблони зацветут, да мы к нулевым трулля и траляля, уже всё оседлаем.

Во втором случае у нас что? Освящаем ракеты? Политота? Американцев не было на Луне?

Что с астрономией в школах? Что с наукой в науке? Вот так это и работает, прогресс это такой очень аморфный вектор, или если угодно маршрут, с которого очень легко сойти в сторону. А если ты про фундаментальные прорывы, то их вообще не спрогнозировать, и для твоих антигравов, нужен буквально переворот. И когда этот переворот будет, и будет ли вообще, никто не знает.
8 662994
>>62990
Почему замедляется прогресс? Какова основная первопричина?
(Я самонадеянно полагаю, что знаю ответ, но хочу послушать твой вариант)
9 662995
>>62994
Потому что тормозит на орбите об остатки атмосферы
10 662996
>>62994
Понятия не имею почему. Нет какой-то единой причины, нет задачи "прогрессировать". Мы помимо "прогрессов" еще хотим жрать, воевать друг с другом, и прочие развлечения устраивать. А перевороты порой происходят вопреки, а не благодаря. Это что-то иррациональное имхо. Дело тут не в политической системе, и дело даже не в финансировании. Эйнштейны появляются рандомно, независимо от чего-либо.
11 662997
>>62996
Двачую. Я появился.
Мимо-новый-эйнштейн
12 662998
>>62996
Можно личный вопрос? Обещаю, не буду троллить и как-либо комментировать. Мне это нужно исключительно для собственного саморазвития, и для рефлексии.
Сколько тебе лет?
13 662999
>>62997
Мальчик, новую теорию относительности нам принеси. Мы к звёздам летать хотим, а не вот это вот всё.
>>62998
24
14 663000
>>62999
>>62998
Я правда не понял какой ты рефлексией страдать собираешься. Да и идиотизм едва ли детерминирован возрастом. Это к вопросу о зумерах и бумерах. Чем отличается батыгин от 60-летнего инженера роскосого? Ну батыгин еще имеет шанс что-то сделать (крайне маленький шанс тащемта), а вот 60-летний инженегр роскосого уже ничего не сделает, зато расскажет про то где снимали полёт на луну, и какие илоны маски пидарасы.
15 663002
Ебанный стыд...
Во-первых, Алькубьерре.
Во-вторых, не упасть, а создавать вокруг корабля изнутри (иначе кина не будет).
В-третьих, НАСА искривляет пространство на десятимиллионную часть, контролируя это сверхточными интерферометрами, до самого варп-привода здесь - как до Антарктиды раком.
16 663003
>>62999
Окей спс.
А теперь мой вариант ответа:
Человек не успевает за прогрессом. Со времён доисторического человека, наш мозг, как утверждают учёные, не изменился. То есть, эйнштейны изобретают теории относительности на том же "железе" что и первые люди, взявшие в руки палку-копалку. Поэтому так всё затормозилось. Технологии вплотную подошли к запилу цивилизации Первого Типа. А хомо сапиенсы не дотягивают. Большинство людей угодило в ловушку, при которой удобство машин вокруг них не усиляет их, а ослабляет. Начиная с фундаментального: мало кто сможет без калькулятора (или гугла) сходу умножить или разделить большие числа. Не говоря о том, чтобы с помощью циркуля и логарифмической линейки что либо матанистое пересчитать. Многими это воспринимается как данность "я тупее дидов, потому что за меня думают машины", "если есть калькулятор, зачем мне помнить матан?".

Френк Герберт предупреждал об этой хуйне полвека назад. Прогресс - это развитие человека, развитие его мозга и его тела, а машины в этом вопросе не должны становиться заменой, а помощью, подспорьем, от которого человек всегда способен отказаться и сделать всё сам, пусть медленнее, но с не менее точно.

И вот из за этой фундаментальной причины, мы и наблюдаем сегодня следствия, от сюра в политике, до деградации в образовании. Учёные знают об этом, но даже говорить на эту тему боятся, потому что решение проблемы - батлерианский джихад.

Вот такую хуйню пишет аноньчик 40 лвл
17 663005
>>63003

>мало кто сможет без калькулятора (или гугла) сходу умножить или разделить большие числа


Словно вот деды все могли.

>решение проблемы


Нейролинк и слияние с машинами. Плюс генетический контроль зачатий. Плюс генная терапия заболеваний. Плюс генная модификация в перспективе.
И да, Хомо Сапиенс саванный африканский, базовая генетическая форма человека обыкновенного для космической цивилизации непригодна. Разве что в форме выходного костюма для отпуска.
18 663007
>>63003
Странно что ты не нашел тут ловушку >>63000
А она там есть. Хокинг теоретизировал одноимёное излучение в 32 года. Эйнштейн "рванул" в 26 лет. Хиггсу же дали нобеля в 84! А первую медальку дали в 52. Я хотя бы поэтому, не вижу причин вообще использовать возраст в качестве аргумента или даже отдельного показателя.
>>63003

>Человек не успевает за прогрессом


Дело скорее всего в том, что ему и не надо, сапиенсу среднестатистическому.

>Со времён доисторического человека, наш мозг, как утверждают учёные, не изменился


У дробышевского было что-то на эту тему, про изменения.

>эйнштейны изобретают теории относительности на том же "железе" что и первые люди, взявшие в руки палку-копалку


И да и нет, проблема в том, что это не железо, вот в чем проблема, с железом проще, а с мясом сложнее.

>Большинство людей угодило в ловушку, при которой удобство машин вокруг них не усиляет их, а ослабляет. Начиная с фундаментального: мало кто сможет без калькулятора (или гугла) сходу умножить или разделить большие числа. Не говоря о том, чтобы с помощью циркуля и логарифмической линейки что либо матанистое пересчитать. Многими это воспринимается как данность "я тупее дидов, потому что за меня думают машины", "если есть калькулятор, зачем мне помнить матан?".


Двачую люто и неистово. Об этом говорит Дугин, дескать-де мы на пороге эпохи деградации, или стагнации, что-то такое.

>Прогресс - это развитие человека, развитие его мозга и его тела, а машины в этом вопросе не должны становиться заменой, а помощью, подспорьем, от которого человек всегда способен отказаться и сделать всё сам, пусть медленнее, но с не менее точно.


Увы, но не всё в этом мире идёт по тому маршруту, который намечался изначально.

>Вот такую хуйню пишет аноньчик 40 лвл


Ну это не хуйня на самом то деле. Но причина едва ли единственная. А решить эту проблему вообще никак. Потому что твоя идея выращивания человека разумного (если ты это подразумеваешь), напарывается на нежизнеспособность, крайне резкую деградацию человека разумного, при изменении условий и нежелание этого самого человека разумного выращивать. Я даже намекать не буду, ты и сам думаю понял о чем я (нынешняя система образования)
18 663007
>>63003
Странно что ты не нашел тут ловушку >>63000
А она там есть. Хокинг теоретизировал одноимёное излучение в 32 года. Эйнштейн "рванул" в 26 лет. Хиггсу же дали нобеля в 84! А первую медальку дали в 52. Я хотя бы поэтому, не вижу причин вообще использовать возраст в качестве аргумента или даже отдельного показателя.
>>63003

>Человек не успевает за прогрессом


Дело скорее всего в том, что ему и не надо, сапиенсу среднестатистическому.

>Со времён доисторического человека, наш мозг, как утверждают учёные, не изменился


У дробышевского было что-то на эту тему, про изменения.

>эйнштейны изобретают теории относительности на том же "железе" что и первые люди, взявшие в руки палку-копалку


И да и нет, проблема в том, что это не железо, вот в чем проблема, с железом проще, а с мясом сложнее.

>Большинство людей угодило в ловушку, при которой удобство машин вокруг них не усиляет их, а ослабляет. Начиная с фундаментального: мало кто сможет без калькулятора (или гугла) сходу умножить или разделить большие числа. Не говоря о том, чтобы с помощью циркуля и логарифмической линейки что либо матанистое пересчитать. Многими это воспринимается как данность "я тупее дидов, потому что за меня думают машины", "если есть калькулятор, зачем мне помнить матан?".


Двачую люто и неистово. Об этом говорит Дугин, дескать-де мы на пороге эпохи деградации, или стагнации, что-то такое.

>Прогресс - это развитие человека, развитие его мозга и его тела, а машины в этом вопросе не должны становиться заменой, а помощью, подспорьем, от которого человек всегда способен отказаться и сделать всё сам, пусть медленнее, но с не менее точно.


Увы, но не всё в этом мире идёт по тому маршруту, который намечался изначально.

>Вот такую хуйню пишет аноньчик 40 лвл


Ну это не хуйня на самом то деле. Но причина едва ли единственная. А решить эту проблему вообще никак. Потому что твоя идея выращивания человека разумного (если ты это подразумеваешь), напарывается на нежизнеспособность, крайне резкую деградацию человека разумного, при изменении условий и нежелание этого самого человека разумного выращивать. Я даже намекать не буду, ты и сам думаю понял о чем я (нынешняя система образования)
16196178929940.jpg203 Кб, 604x604
19 663009
как добиться земного комфорта (состав/плотность атмосферы; жидкая вода) на других планетах, которые много больше или меньше земли?
E3s8SbuVcAEQcS.jpg119 Кб, 1536x864
20 663014
Вот я слышал про ядерные детанационные двигатели, которые взрывают ядерные бомбы. А можно ли использовать другую обычную взрывчатку как топливо? Например капсулы гексогена какие-то взрывать за аппаратом? Допустим создать что-то типа маленьких таблеток взрывчатки, выбрасывать их сзади спутника, поджигать лазером и ускоряться? Без всякой ядерной вонючки.
21 663015
>>63014
можно, но это будет менее эффективно чем обычные ракетные двигатели
22 663016
>>63015
Насколько неэффективно? Думаю это можно было бы для корректировки астероидов использовать. В таком случае на астероиде просто устанавливаем что-то типа тазика на поверхность близко к центру тяжести и нужного угла вращения, в тазик кладём капсулы взрывчатки и детонируем. Взрывчатка из таза создаёт импульс.
Можно даже тазик нахуй выкинуть, просто ямы на астероидах рыть нужной формы, запихивать в них взрывчатку и так можно останавливать вращение, менять орбиту астероидов. Ты скажешь нужно дохуя чтобы орбиту поменять, но так можно спланировать тонко и на десятилетия вперёд рассчитать гравитационные манёвры. Чуть чуть подтолкнул астероид к пролёту мимо Венеры, потом пукнул в перицентре, чуть пукнул в другом месте, потом пукнул у орбиты Марса и бац - уже рядом с Землей-Луной, тут пук помощнее и еще пара пуков и на орбите Земли.
23 663023
>>63016

>тут пук помощнее и еще пара пуков и на орбите Земли.


Никто не позволит пукать рядом с Землей. Даже если у тебя суперкомпьютер, который все идеально рассчитает, астероиды можно использовать как ОМП в войнах и терактах.
24 663024
>>63016
Ракетный двигатель уже по своей сути устройство направленного взрыва. Только взрыв происходит непрерывно, а сопло позволяет эффективно направлять его, минимизируя потери энергии и тяги.
25 663043
>>63016
Эту хуйню про взрывчатку уже пора в шапку добавлять
1624630402507.jpg75 Кб, 1080x286
26 663051
>>62985
Пока что не удается сжижить вакуум до нужной величины
27 663052
>>62987
Да тем, что непонятно, что там внутри и как работает.
Алхимики тысячу лет пытались получить из свинца золото путем химических реакций, а надо было проникнуть глубже, в ядра и атомы.
Теперь нам нужно проникнуть ещё глубже, чтобы постичь гравитацию.
Да и не только ее, саму суть пространства и времени.
28 663056
>>63014
Вся суть взрыволёта - сделать хотя бы тупую орочью WAAAAGH-колесницу с огромными потерями на распыление говен во все стороны, и перетяжелённую из-за щита, ибо сопло для этой хуйни ты не сделаешь даже из анобтаниума.

Для гексогена же это смысла никакого не имеет, потому что 1) энергию рабочего тела, разогретого химической реакцией, и без подобных плясок можно направить в нужную сторону оптимальным соплом, и 2) водород-кислород всё равно лучше чем гексоген. Гексоген это ВВ, он имеет высокую скорость детонации и бризантность, он хорошо заточен под разрушение любой хуйни, рядом с которой взрывается. А ракетное топливо из ВВ как из говна пуля.
29 663057
>>662686 →

>Поэтому надо на обратной стороне Луны заебашить йоба-телескоп


Это всё равно что в открытом космосе, только сложнее, дороже и побъёт метеоритами. Смысла нет.
Был только недавно интересный концепт - растянуть роверами над лунным кратером радио-отражающую сетку, и получить таким образом большой зенитный радиотелескоп вроде Аресибо или FAST, обойдясь относительно малыми средствами. Это ещё куда ни шло идея. А зеркала туда слать - смысла мало.
30 663153
>>63009

> меньше


закинуть туда бочку мкс

> много больше


уже нахватались газиков до невозможности
31 663211
>>63016

>Насколько неэффективно?


Раз в десять.
32 663224
хуле жарко то так?
33 663225
>>63224
Потому что Земля сейчас ближе к солнцу чем обычно.
34 663226
>>63225
чел, всё наоборот, мы дальше чем обычно, нормально отвечай давай
35 663227
>>63226
Про наклон земной оси в школе же должны рассказывать. И сейчас наши широты сильнее нагреваются солнцем чем в зимний период. Я на троллинг ответил тупостью троллингом.
36 663229
А в каких школах и институтах вы учились, что так хорошо поясняете за химию, физику, астрофизику?
37 663245
>>63229
Два года на дваче сделают любого астрофизиком, сынок.
38 663255
>>63245
Я тут больше 12 лет. Что-то меня двач сделал социольным лузером и женоненавстником, который зависим от порно.
39 663277
>>63255
Свинья везде грязь найдет
Наземный астероид майнинг 40 663411
Допустим мы научились маневрировать астероидами задёшего.

Можно ли так подобрать параметры орбиты и скорость астероида (богатый металлами, размером со стадион например) что бы более-менее мягко "посадить" его на землю?

Как-нибудь типа догонным курсом со скоростью чуть больше чем скорость земли, по касательной затормозить об атмосферу и шмякнуться где-нибудь в пустыне?

Что бы не взорвался, не устроил землетрясений или экологических неприятностей,
может быть развалился на несколько крупных кусков и почти не сгорел в атмосфере,
что бы можно было его потом дёшево найти и собрать в пустыне и поживиться всяким так редкоземом.
41 663415
>>63411
Хуй. Минималка на входе в атмосферу 11 км/с.
Хочешь целиком металлов - пили на орбите и спускай на шаталах или строй неракетные способы вроде пусковой петли или скайхука (которому как раз и надо спускать вещи с орбиты чтобы самому на ней оставаться).
42 663419
>>63415

>Минималка на входе в атмосферу 11 км/с.


Но почему? У нас нет задачи крутится вокруг земли.

Земля ебошит вокруг солнца 29,78 км/c
Пускаем астероид по касательной к орбите земли со скоростью
29,XX км/c и подгадываем что бы земля и астероид каждый были в одно и то же время в точке пересечения орбиты земли и траектории астроида.

Таким образом когда астероид оказывается в окресностях земли - вектора его и земли почти совпадают, разница скоростей 29,XX -29,78 = +-0,YY кмс

Я не знаю как выгоднее, что бы астероид чуть спешил или чуть отставал или имел ровно ту же скорость, но тут уже вовсю работает гравитация земли, которая тянет гостя к себе гася разницу скоростей, т.е. это по сути свободное падение, можно хоть с нуля кмс начинать падатьб если полностью совпадают скорости земли и астероида.
43 663422
>>63415
Это как если ОЧЕНЬ быстро ехать на мотоцикле, я тебе в спину будут стрелять и ты такой оборачиваешься, а за спиной висят летящие тебе вслед пули и ты их такой ручонкой своей собираешь и в карман, потому что разница скоростей твоей и и пули 2 см в минуту.
44 663437
>>63419

>Таким образом когда астероид оказывается в окресностях земли -


он оказывается в гравитационном колодце Земли и за счет ускорения вызванного её притяжением и наберёт эти 11 километров в секунду относительно Земли.
Тебе надо припарковать его на орбите, для этого надо оттормозиться, чтобы он не улетел, совершив гравитационный маневр, или не жахнулся в землю со скоростью 11 кмс.

>>63422
А теперь представь что у тебя на спине йоба-магнит, и пули магнитящиеся. Пули едва приближались к тебе, но за счета магнита ускорились и порешали тебя. У Земли этот "йоба-магнит" - сила притяжения.
45 663460
46 663477
>>63255
Надо было в быдлятню и порашу не ходить.
47 663493
>>62989
Будешь говночистом в цифровом гулаге. Вот тебе и весь научный прогресс.
48 663513
>>63411
нужно вырезать из астероида каменный планер чтоб он сам сел на землю
49 663557
Легко ли обнаруживать черные дыры планетарной массы?

Это может быть разгадкой парадокса Ферми: в ходе научных исследований цивилизация создаёт черную дыру, в которую засасывает их планету.
50 663573
>>63557
Сейчас практически нереально.
51 663576
>>63557
Существуют гораздо более правдоподобные теории, например что они тупо прекращают размножаться предпочитая гедонизм. Посмотри на самые развитые нации на Земле двачеров, они потихоньку вымирают.
52 663577
>>63557
Довольно трудно. Только если она будет рядом и будет подсвечиваться чем-нибудь
Чем меньше ЧД, тем хуже на нее аккреция. В добавок ЧД меньше определенного порога имеют тенденцию выбрасываться из гравитационного связных систем, в том числе из Галактики.
Микроскопические ЧД вообще не способны поглощать вещество, так что вряд ли моченые ксеносов могут так устроить себе экстерминатус.
53 663615
>>63513
Ага, плавить металл на орбите, штамповать полые сферы а-ля бензобаки от ракет, и кидать вниз. Практика показывает что они долетают норм, не сгорают, только заебёшься их потом искать.
54 663636
>>63615
На австралию дропать надо, там удобно.
55 663645
Пришел в голову, наверное, самый тупой вопрос.

Имеем планеты - они вращаются вокруг своей оси. Вращение это замедляется.

Есть планеты в состоянии приливного захвата (у нас таких нет, но вообще бывают, у нас есть такие спутники).

Для приливного захвата нужно, чтобы планета находилась достаточно близко к звезде.

Вопрос - а может ли планета просто прекратить вращаться и остановиться? Где-нибудь на месте Земли, Марса или Нептуна, просто вращаться, вращаться и перестать, остановившись одной стороной к звезде? Или сформироваться как-нибудь так, чтобы не вращаться?
56 663647
>>63645
Может.
57 663657
Сколько проживет человек земных лет на другой планете с земными условиями?
К примеру построили базу на Марсе Ганимеде с теплицей, для организма человека все хорошо, период оборачиваемости вокруг Солнца дольше чем на Земле, человек проживет дольше или меньше если отсчёт вести в земных годах с его рождения? Увеличится ли продолжительность жизни на планете с большим периодом обращения вокруг солнца или уменьшится? Тогда с более быстрым обращением вокруг звёзды может больше времени на биологическую жизнь будет дано?
58 663660
>>63657
При тех же условиях ничего не должно поменяться. При меньшей гравитации проживет может дольше, т.к. на сердце нагрузка меньше. При большей радиации проживет меньше. При меньшей инсоляции проживет меньше и так далее.
59 663662

> the acceleration of a test body at the event horizon of a black hole turns out to be infinite in relativity.


Если так, поч пишут, что при падении в смчд пересечении гс ты не заметишь и будешь просто продолжать ускоренно падать в сингулярность? Раз ускорение бесконечно, то и ссингуляришься ты разом, не?
60 663669
>>63662
откуда эта шиза вообще взялась? ладно приливные силы, нашли граничный случай, но фотонная сфера, градиент дельта тэ, асимптота спейстайма и ограничения для переносчиков полей куда делись? что за ебланы это вбрасывают?
61 663671
>>63669
Тогда по-другому: если g на горизонте и под ним конечно, то что помешает нам сунуть туда сверхно не бесконечнопрочный стержень и высунуть обратно (пусть и с гигантскими усилиями)?
62 663676
>>63671
У тебя при пересечении ГС теряются молекулярные связи. Атом на этой стороне ГС не контачит с атомом на той стороне, фотоны ЭМ взаимодействия не доходят друг до друга и у тебя получается что палка кончается на ГС.
63 663677
>>63671
Дурачок, из-под горизонта черной дыры ничто не может выйти, даже свет, потому что для преодоления фантастически сильного искривления пространства-времени под горизонтом нужна сверхсветовая скорость. Ты со сверхсветовой как собрался свой стержень ебучий высовывать?
64 663680
>>63677
Да, не может, именно потому что g под гс бесконечно и даже минимальное отдаление от сингулярности невозможно. Но это возвращает к первоначальному моему вопросу.
>>63676
Поскольку ото похуй на квантмех, можно не брать в расчёт, что тела состоят из частиц и обмениваются частицами взаимодействия,- а вместо этого вообразить сплошные непрерывные тела со скоростью звука (передачи взаимодействия), ограниченной световой. Например, есть ракета, наполовину погружённая под гс; если g не бесконечно, то пусть ракета высрет топливо с тягой больше, чем это g. Тогда она вылетит. Либо g бесконечно - тогда никак не вылетит. Но тогда возвращаемся к первому вопросу. Сложно это для меня, короче.
65 663686
>>63680

>можно не брать в расчёт, что тела состоят из частиц и обмениваются частицами взаимодействия


Объясни почему, я не понимаю.
66 663693
>>63686
А с чего бы, собсна, ТО полностью независимо от КМ развивалась. Лимитированность всех взаимодействий скоростью света, красное смещение как эффект доплера - до квантов открыты были.
Да пусть даже тела состоят из атомов в доквантовом понимании - но тогда эти атомы суть классические частицы, с размером, ну и пускай неразрушимостью, что тоже допускалось в классике. Но тогда можно пол-атома под ГС сунуть, ну и тд. Это уже в КМ завезли дуализмы, нелокальность, точечность, квантование и прочие нёхи.
67 663699
>>63680

> вообразить


значение знаешь?
68 663700
>>63699
бультерьер
69 663701
>>63693

>Но тогда можно пол-атома под ГС сунуть


Ты только что расщепил атом.
70 663726
буду ли я космохуй?
71 663748
>>63701
Я имел в виду атом 19 века, т.е. буквально неделимый, но имеющий размер и чёткое место/импульс. Тогда ж их воображали неразрушимыми шариками или чем-то подобным.
72 663797
Для того чтоб вылететь из-под горизонта событий нужно двигаться быстрее скорости света. А можно ли туда залететь на Альбукерке? А потом вернуться обратно?
73 663800
>>63797
Думаю потребуется энергия нескольких наших вселенных для такого маневра.
74 663838
>>63797
Нельзя, потому что все твои хуипиздерки - ёбаные маняфантазии, существующие лишь в воображении всяких шизов.
75 663841
>>63838

>хуипиздерки


Как мы ловко не отходя от двача открываем новые классы объектов.
76 663860
Первый раз в разделе в связи с появившимся неожиданно вопросом. А почему официально не запускают всякие мини ракеты размером с бутылку на орбиту, в космос, на солнце и тд. Если запускают почему не афишируют? В связи с посещением раздела увидел тред где делают ракеты самодельные примерно такого размера, но похоже выше крыши соседней многоэтажки они не взлетают особо, а что мешает дома собрать вот такую миниракету только по всем законам правильно и запустить ее куда-нибудь в космос? Раз размер меньше, то очевидно топлива мало надо и скорость выше от малого веса будет. Неужели невозможно это сделать без средств самодельно в гараже или на даче в таком мини размере? Сейчас что только не собирают дома, и компьютеры и роботов всяких. В теме вообще не шарю, но интересно как и почему, объясните всё-всё если не лень.
77 663861
>>63657
Все просчитать не получится, что-нибудь окажется не так как должно быть, а за помощью далеко лететь. Куча тестов и добровольцев будут все это определять годами, десятками или даже сотнями лет жертвуя жизнями. Все что угодно может отрицательно повлиять. А положительное влияние очень маловероятно.
78 663862
>>63860
Закон квадрата-куба. Уменьшая ракету, полезная нагрузка уменьшается непропорционально размеру. Самые малые ракеты делает Япония, самая мелкая из них 10 метров на полметра диаметра, весит 2,5 тонны, выводит всего 4кг на орбиту. Меньшая ракета до орбиты даже не долетит. Размером с бутылку возможно и линии Кармана не преодолеет. Такая вот хуйня
image.png105 Кб, 262x192
79 663866
>>63862

>Размером с бутылку возможно и линии Кармана не преодолеет.


Но бутылки обычно больше карманов.
80 663873
>>63860
Корейская Perigee Aerospace хочет запускать ракету Blue Whale 1 общей массой всего 1.8 тонн, и ПН на солнечно-синхронную 50кг. Это будет самая маленькая ракета в мире, если не обосрутся.

Любителям это недоступно, ибо ЖРД и тем более криогенные компоненты (у корейцев жидкий кислород и жидкий метан) это охуеть какой технологический уровень, а на твердотопливных ракета выходит большой - нужен 9 с хером км/с запас скорости. Японская твердотопливная Lambda 4S в семидесятых весила 10 тонн, для сравнения. Плюс требуемая инженерия системы управления, отслеживания, телеметрии - требуют экспоненциально растущих затрат.

В общем, среди любителей поднять ракету на 1км может и школьник. Поднять на 10км - это уже хороший любительский уровень. Поднять на 35км могут единицы. https://www.youtube.com/watch?v=sQw_C5KLhFM Коснуться линии Кармана - вообще только Copenhagen Suborbitals только и могут, от любителей у них только название, там полноценное микро-КБ, всё серьёзно.

Компании же не пускают мелкие ракеты, ибо невыгодно: чем больше ракета, тем дешевле её пускать. Даже Маск в своё время (2004-2006) обломал зубы с Фалконом 1, посчитав его нежизнеспособным при такой цене за кило - потенциальные покупатели бы пускали мелкоспутники на тяжёлых попутках. Лишь несколько лет назад образовался рынок для мелких ракет, тема потихоньку двигается.
81 663875
>>63862

>самая мелкая из них 10 метров на полметра диаметра, весит 2,5 тонны, выводит всего 4кг на орбиту


Это Лямбда-4S, как раз та что весит 10 тонн, а не 2.5. NOTS-EV-1 весила 2.5 и могла в 10км/с дельты, но у неё был ЖРД и воздушный старт. (и в своё время на орбиту так и не вышла)
82 663877
>>63862

>Размером с бутылку возможно и линии Кармана не преодолеет.


>возможно


https://www.youtube.com/watch?v=bDoh8zQDT38
SL-10 это самый близкий к любительской суборбите аппарат. Строго говоря не любители, но очень близко по уровню.
(кстати если бы не было стабилизации вращением, то задача на порядок бы усложнилась - к вопросу о любительских запусках)
83 663878
>>63875
я про SS-520-4
>>63877

>Aerospace Inc. launched the 20-foot (6 meter) tall SL-10


Нихуёвые у тебя бутылки.

Смотрю вы в залупу полезли, ладно. Суть >>63860 поста была в том, что есть нижний предел размера ракеты. Помимо фактора закона квадрата-куба, который приближает ПН к нулю квадратично от линейного размера, есть ещё фактор массового совершенства, т.к. толщину стенок нельзя уменьшать пропорционально, сохраняя прочность. Этот фактор линейный, и пробивает дно при определенном размере, делая в принципе невозможным постройку оче малых ракет, которые выведут на орбиту миллиграмм
84 663879
>>63878

>Нихуёвые у тебя бутылки.


Я про это и пытался сказать, в общем-то. Бутылка хуй куда долетит.
85 663883
>>63878

>я про SS-520-4


Это SS-520-5. (SS-520, вариант 5)
Кстати довольно близко к любительскому уровню, если подумать. Даже фейл в 4 модификации у них вышел довольно детский, не был термоизолирован кабель-канал или что-то такое, если правильно помню.
86 663887
>>63883
У них даже стабилизация вращением. На орбитальной ракете, Карл!
87 663890
Кстати, такой вопрос. Все суборбитал мелкоракеты, пиздошат на 6ж или больше со старта, как будто они зенитные. Вопрос, нахуя такой эксессив траст? Этож уменьшает потолок излишним трением воздуха, и сделав двигатель слабее и легче можно больше дельты получить. В чём выгода такого старта?
88 663896
>>63890
Ты хуйни какой-то начитался. Гравитационные потери во много раз больше аэродинамических, чем быстрее летит ракета, тем меньше потерь на борьбу с гравитацией, которая сжирает 9.8 м/с дельты каждую секунду полёта. Для сравнения, аэродинамические потери обычно составляют <100 м/с за весь полёт.

Суборбитальные ракеты все на твёрдом топливе, там нет никакого двигателя, большая тяга у них бесплатная. Орбитальные жидкостные ракеты так не могут, там более мощные двигатели действительно делают ракету тяжелее, и что намного важнее, сильно дороже.
89 663897
Вращение планеты частично компенсирует её гравитацию?

Если взять две идентичные планеты, одна вращается, вторая нет
- они будут разного размера?

Та что вращается будет больше, так как её плотность будет меньше из-за центробежного ускорения?

Если ебануть по планете вскользь чем-то сопоставимо тяжёлым - возможно ли заметно ускорить её вращение? Если удар был в том же направлении куда она уже вращалась.

Если геологически активная планета ускорит вращение и её из-за этого немного раздует, то её кора вероятно потрескается на отдельные крупные куски, очертания которых будут совместмы друг с другом?

Кто-то уже спекулировал таким способом на тему континентов земли и столкновения с какой-нибудт там тейей?
90 663902
>>63896

>Гравитационные потери во много раз больше аэродинамических


Это верно только для больших ракет. У совсем мелких аэродинамические не пренебрежимо малы. Кроме того, небольшая часть затрачивается на обтюрацию, т.к. у них всех стабилизация вращением. Но баланс между этими потерями находится на ебовых ускорениях всё равно, да. Даже у мелких.
91 663903
>>63887
А че не так?
92 663904
>>63897
Да
Да
Нет
Да
Нет
Нет
93 663905
>>63897

> Вращение планеты частично компенсирует её гравитацию?


Да, конечно. У Земли на экваторе g меньше чем на полюсах. Есть звёзды, которые вращаются настолько быстро, что их внешние слои находятся на грани распидораса. https://en.wikipedia.org/wiki/VFTS_102
Есть даже классический НФ роман про жизнь на планете с сильно разным ускорением падения на экваторе и полюсах, когда путешественники нелюдской расы хуячат на полюс и наблюдают экзотические эффекты от увеличенного веса - забыл как звать.

>Если взять две идентичные планеты, одна вращается, вторая нет - они будут разного размера?


>Та что вращается будет больше, так как её плотность будет меньше из-за центробежного ускорения?


>Если ебануть по планете вскользь чем-то сопоставимо тяжёлым - возможно ли заметно ускорить её вращение? Если удар был в том же направлении куда она уже вращалась.


На все вопросы ответ да. Надо помнить, что планету распидорасит вращением задолго до того, как скорость вращения экватора достигнет первой космической - т.к. её распирает изнутри. Есть какой-то лимит, не помню какой, что-то вроде трети скорости.

>Если геологически активная планета ускорит вращение и её из-за этого немного раздует, то её кора вероятно потрескается на отдельные крупные куски, очертания которых будут совместмы друг с другом?


Да, но вообще всё это должно потом устаканиться в геологических масштабах времени.

>Кто-то уже спекулировал таким способом на тему континентов земли и столкновения с какой-нибудт там тейей?


Есть теория что Луна образовалась именно таким способом, выплеснуло кусок говна от столкновения Земли с сопоставимым телом.
detail692c4a461e977f0c86594e3b7abf0adf.jpg74 Кб, 600x400
94 663906
95 663908
>>63903
Вращение это больше фишка простеньких одноступенчатых суборбитальных ракет - т.к. им не надо разделяться или поворачивать. Орбитальным надо, а вращение всё усложняет - поэтому они обычно имеют сложную СУ и подвес качания сопла/движка.
96 663920
>>63905
НФ роман называется Mission of Gravity (Экспедиция «Тяготение»).
97 663961
Мне кажется или Сергей Борисович немного поддатый на этой лекции?

https://www.youtube.com/watch?v=bcXbBO6JSkQ
98 663975
Какая скорость звука в космосе?
100 664026
>>63984

>in_plasma


А чо сразу плазма? Облако холодного молекулярного водорода с плотностью 1000 молекул на кубический сантиметр. Какая там будет скорость звука?
101 664027
>>64026
Никакая. Это вакуум, чисто механическая передача невозможна - длина свободного пробега молекул больше чем расстояние между ними. В плазме ещё хотя бы возможно некое магнитоакустическое подобие звука, если немного натянуть сову на глобус.

Но ударная волна как ни странно возможна - если некое тело будет ебошить достаточно быстро, чтобы сгребать перед собой молекулы и создавать волну сжатия.
102 664028
>>64027

>Никакая


Ясно. Понятно.
103 664029
>>64027
Комос полон ударных волн, если есть ударная волна в среде, значит в среде есть скорость звука.
104 664037
>>63975
1000 м/с
10e689abcda30ce453a6725dd9e9ccec[1].png239 Кб, 1636x1254
105 664092
Привет космач. Помнится мне, как-то тут всерьёз обсуждался пикрил про мужика, который просто с ракеты упал.
Казалось бы, порешали, что если бы по какой-то причине мужику было нужно срочно спуститься с МКС на Землю, и он решил бы спрыгнуть с МКС и упасть вниз, то ему помешало бы два фактора:
1) нужно сбить 7,8км/с до 0, чтобы вместо движения по орбитоиде и зажаривания аэроторможением с потерей времени, падать строго тупа вниз
2) придумать способ затормозить, чтобы в самых низах атмосферы активировать простой парашют

Насколько я помню, тогда сказали, что если 1 сложно, но можно, при условии, что у нас именно сам мужик, без нанокапсулы или посадочного модуля размером с автомобиль Ока, а вот про 2 писали, что мужик сгорит, замёрзнет и порвёт все стропила при попытке тормознуть парашютом, ибо трение воздуха и ускорение свободного падения.
Но я для себя вот что узнал:

>Скорость падения тела в газе или жидкости стабилизируется по достижении телом скорости, при которой сила гравитационного притяжения уравновешивается силой сопротивления среды.


>Согласно законам механики Ньютона, тело, находящееся в состоянии свободного падения, должно двигаться равноускоренно, поскольку на него действует ничем не уравновешенная сила земного притяжения. При падении тела в земной атмосфере (или любой другой газообразной или жидкой среде) мы, однако, наблюдаем иную картину, поскольку на сцену выходит еще одна сила. Падая, тело должно раздвигать собой молекулы воздуха, которые противодействуют этому, в результате чего начинает действовать сила аэродинамического сопротивления или вязкого торможения. Чем выше скорость падения, тем сильнее сопротивление. И, когда направленная вверх сила вязкого торможения сравнивается по величине с направленной вниз гравитационной силой, их равнодействующая становится равной нулю, и тело переходит из состояния ускоренного падения в состояние равномерного падения. Скорость такого равномерного падения называется предельной скоростью падения тела в среде.


>Модуль предельной скорости падения зависит от аэродинамических или гидродинамических свойств тела, то есть, от степени его обтекаемости. В самом простом случае идеально обтекаемого тела вокруг него не образуется никаких дополнительных завихрений, препятствующих падению, — так называемых турбулентностей — и мы наблюдаем ламинарный поток. В ламинарном потоке сила сопротивления вязкой среды возрастает прямо пропорционально скорости тела. Вокруг мелких дождевых капель в воздухе, например, образуется классический ламинарный поток. При этом предельная скорость падения таких капель будет весьма мала — около 5 км/ч, что соответствует скорости прогулочного шага. Вот почему моросящий дождь порой кажется «зависшим» в воздухе. Еще меньшую предельную скорость имели масляные капли, использованные в опыте Милликена.


>При движении в вязкой среде более крупных объектов, однако, начинают преобладать иные эффекты и закономерности. При достижении дождевыми каплями диаметра всего лишь в десятые доли миллиметра вокруг них начинают образовываться так называемые завихрения в результате срыва потока. Вы их, возможно, наблюдали весьма наглядно: когда машина осенью едет по дороге, засыпанной опавшей листвой, сухие листья не просто разметаются по сторонам от машины, но начинают кружиться в подобии вальса. Описываемые ими круги в точности повторяют линии вихрей фон Кармана, получивших свое название в честь инженера-физика венгерского происхождения Теодора фон Кармана (Theodore von Kármán, 1881–1963), который, эмигрировав в США и работая в Калифорнийском технологическом институте, стал одним из основоположников современной прикладной аэродинамики. Этими турбулентными вихрями обычно и обусловлено торможение — именно они вносят основной вклад в то, что машина или самолет, разогнавшись до определенной скорости, сталкиваются с резко возросшим сопротивлением воздуха и дальше ускоряться не в состоянии. Если вам доводилось на большой скорости разъезжаться на своем легковом автомобиле с тяжелым и быстрым встречным фургоном и машину начинало «водить» из стороны в сторону, знайте: вы попали в вихрь фон Кармана и познакомились с ним не понаслышке.


>При свободном падении крупных тел в атмосфере завихрения начинаются практически сразу, и предельная скорость падения достигается очень быстро. Для парашютистов, например, предельная скорость составляет от 190 км/ч при максимальном сопротивлении воздуха, когда они падают плашмя, раскинув руки, до 240 км/ч при нырянии «рыбкой» или «солдатиком».



Значит, если мы берём в расчёт не редбульного прыгуна с прыжком десяточек км, а полноценные 410км орбитальной высоты МКС, то и тут, получается, выжить при прыжке вниз не проблема. Нужно просто как-то охлаждаться и потом сбросить 190км/ч.
10e689abcda30ce453a6725dd9e9ccec[1].png239 Кб, 1636x1254
105 664092
Привет космач. Помнится мне, как-то тут всерьёз обсуждался пикрил про мужика, который просто с ракеты упал.
Казалось бы, порешали, что если бы по какой-то причине мужику было нужно срочно спуститься с МКС на Землю, и он решил бы спрыгнуть с МКС и упасть вниз, то ему помешало бы два фактора:
1) нужно сбить 7,8км/с до 0, чтобы вместо движения по орбитоиде и зажаривания аэроторможением с потерей времени, падать строго тупа вниз
2) придумать способ затормозить, чтобы в самых низах атмосферы активировать простой парашют

Насколько я помню, тогда сказали, что если 1 сложно, но можно, при условии, что у нас именно сам мужик, без нанокапсулы или посадочного модуля размером с автомобиль Ока, а вот про 2 писали, что мужик сгорит, замёрзнет и порвёт все стропила при попытке тормознуть парашютом, ибо трение воздуха и ускорение свободного падения.
Но я для себя вот что узнал:

>Скорость падения тела в газе или жидкости стабилизируется по достижении телом скорости, при которой сила гравитационного притяжения уравновешивается силой сопротивления среды.


>Согласно законам механики Ньютона, тело, находящееся в состоянии свободного падения, должно двигаться равноускоренно, поскольку на него действует ничем не уравновешенная сила земного притяжения. При падении тела в земной атмосфере (или любой другой газообразной или жидкой среде) мы, однако, наблюдаем иную картину, поскольку на сцену выходит еще одна сила. Падая, тело должно раздвигать собой молекулы воздуха, которые противодействуют этому, в результате чего начинает действовать сила аэродинамического сопротивления или вязкого торможения. Чем выше скорость падения, тем сильнее сопротивление. И, когда направленная вверх сила вязкого торможения сравнивается по величине с направленной вниз гравитационной силой, их равнодействующая становится равной нулю, и тело переходит из состояния ускоренного падения в состояние равномерного падения. Скорость такого равномерного падения называется предельной скоростью падения тела в среде.


>Модуль предельной скорости падения зависит от аэродинамических или гидродинамических свойств тела, то есть, от степени его обтекаемости. В самом простом случае идеально обтекаемого тела вокруг него не образуется никаких дополнительных завихрений, препятствующих падению, — так называемых турбулентностей — и мы наблюдаем ламинарный поток. В ламинарном потоке сила сопротивления вязкой среды возрастает прямо пропорционально скорости тела. Вокруг мелких дождевых капель в воздухе, например, образуется классический ламинарный поток. При этом предельная скорость падения таких капель будет весьма мала — около 5 км/ч, что соответствует скорости прогулочного шага. Вот почему моросящий дождь порой кажется «зависшим» в воздухе. Еще меньшую предельную скорость имели масляные капли, использованные в опыте Милликена.


>При движении в вязкой среде более крупных объектов, однако, начинают преобладать иные эффекты и закономерности. При достижении дождевыми каплями диаметра всего лишь в десятые доли миллиметра вокруг них начинают образовываться так называемые завихрения в результате срыва потока. Вы их, возможно, наблюдали весьма наглядно: когда машина осенью едет по дороге, засыпанной опавшей листвой, сухие листья не просто разметаются по сторонам от машины, но начинают кружиться в подобии вальса. Описываемые ими круги в точности повторяют линии вихрей фон Кармана, получивших свое название в честь инженера-физика венгерского происхождения Теодора фон Кармана (Theodore von Kármán, 1881–1963), который, эмигрировав в США и работая в Калифорнийском технологическом институте, стал одним из основоположников современной прикладной аэродинамики. Этими турбулентными вихрями обычно и обусловлено торможение — именно они вносят основной вклад в то, что машина или самолет, разогнавшись до определенной скорости, сталкиваются с резко возросшим сопротивлением воздуха и дальше ускоряться не в состоянии. Если вам доводилось на большой скорости разъезжаться на своем легковом автомобиле с тяжелым и быстрым встречным фургоном и машину начинало «водить» из стороны в сторону, знайте: вы попали в вихрь фон Кармана и познакомились с ним не понаслышке.


>При свободном падении крупных тел в атмосфере завихрения начинаются практически сразу, и предельная скорость падения достигается очень быстро. Для парашютистов, например, предельная скорость составляет от 190 км/ч при максимальном сопротивлении воздуха, когда они падают плашмя, раскинув руки, до 240 км/ч при нырянии «рыбкой» или «солдатиком».



Значит, если мы берём в расчёт не редбульного прыгуна с прыжком десяточек км, а полноценные 410км орбитальной высоты МКС, то и тут, получается, выжить при прыжке вниз не проблема. Нужно просто как-то охлаждаться и потом сбросить 190км/ч.
106 664093
>>64092

>Нужно просто как-то охлаждаться и потом сбросить 190км/ч.


Молодец, ты изобрёл вхождение в атмосферу. Теперь придумай, как ты будешь "просто охлаждаться" "без нанокапсулы или посадочного модуля размером с автомобиль Ока".
107 664095
>>64093
А обмазаться говном не поможет ли?
По консистенции как пластилин - просто обмажься. Термопроводность низкая, с отрывом баребухов и комочков от общего говномассы комки будут забирать с собой тепло. По мере падения воздуху будет тереть фекальную обшивку, стирая её и тем самым охлаждая.
Ну или обмазаться барсучьим или тюленьим жиром. Если жир у зверей, живущих на северах спасает их от холода месяцами, то может и от жара пару минут выдержит. На вскидку, 5см слоя обмазывания жиром и говном помогут тебе не сгореть в атмосфере.
108 664101
>>64095

>термозащита из говна


2ch Space Program протёк.
109 664143
>>64029

>если есть ударная волна в среде, значит в среде есть скорость звука


Неправильный ответ. Это не скорость звука, это параметры (не скорость!), при которых образуются ударные волны. Сам по себе звук в классическом смысле в облаке молекулярного водорода передаваться не может - оно для этого слишком разрежено.
110 664144
>>64092
Это всё для дозвука. Уже редбульный прыгун на сверхзвуке летел, а с орбиты ты будешь лететь на глубоком гиперзвуке, где доминируют аэротермодинамические эффекты. Режим полёта совершенно другой.

>>64095
Ну вот, теперь ты изобрёл абляционную защиту. Только там нагрев такой, что отрыв кусочков не поможет (хотя теоретически реально возить с собой вагон замороженной мочи и во время спуска кипятить её, выдавая струю пара, просто мочи надо дохуя).

Нормальная абляционная защита по-другому работает - баребухи разлагаются на пердёж и создают газовую оболочку, непрозрачную для инфракрасного излучения. Волна сжатия бежит впереди космонавта и лучирует в него, нагревая, а оболочка пердежа не даёт нагреть, отражая почти всё обратно.

Но для абляционной защиты всё равно капсула нужна, ибо даже с абляционкой нагрев такой что сваришься. Без капсулы приземлиться можно - только нужен низкоэнергетический спуск. Для этого надо:
1) входить в атмосферу очень полого, размазывая торможение по длинной траектории
2) увеличить площадь торможения до очень большой, относительно массы тела (увеличить баллистический коэффициент)
Площадь увеличить можно двумя способами - либо раскрыв перед собой огромный зонтик (метров 10, для примера). И падать как лягушка на листе. Либо размотать за собой бобину тонкого стального троса, километров на 20 - и держаться за него. Чтобы он тебя не догнал, надо вовремя отцепиться.
В обоих случаях говном и жиром обмазаться не мешает, жарко будет всё равно.
111 664145
>>64143

>Это не скорость звука


Ударная волна в среде есть, а скорости звука в среде нет? Ебать ты смешной.
112 664146
>>64145
Вот такие дела, представляешь? Нет звука - нет у него скорости
113 664147
>>64145

>Ударная волна в среде есть


Да, дополню. Ударные волны в молекулярных облаках от гиперскоростных объектов, как и атмосферы комет и прочие нестабильные явления - известны как "экзотические атмосферы", это целое направление.

Причём передача звука в плазме и ударные волны от звезд и черных дыр - явление совсем другого рода, не чисто механическое
114 664148
>>64147

>от гиперскоростных объектов


>гиперскоростных



Скорость звука в среде с температурой 7 000 - 10 000K (средняя температура межзвездной среды) и межзвездной полностью примерно 10 км/c - это ты считаешь гиперпуперскоростями?

>передача звука в плазме и ударные волны от звезд и черных дыр - явление совсем другого рода



Ударные волны в межзвездной среде есть, а скорости звука нет?
115 664149
>>64148

>Ударные волны в межзвездной среде есть, а скорости звука нет?


Челик видать не понимает что такое ударная волна.
116 664150
>>64148

>Ударные волны в межзвездной среде есть, а скорости звука нет?


Зачем ты переспрашиваешь? Я ведь уже ответил. Да, ударная волна есть, а скорости звука нет, потому что нет самого звука в принципе. Вот такие чудеса. Ударная волна образуется только в присутствии объекта, который сгребает молекулы, и это не звук и никуда он не распространяется за пределами волны, это именно объект создаёт перед собой другую среду, отличную от окружающей.

>Скорость звука в среде с температурой 7 000 - 10 000K


Тебе уже сказали что про плазму речь не идёт, там механизм совершенно иной, и в плазме звук возможен независимо от того насколько разрежена среда.

В целом я не спорю, я просто отвечаю на тупой вопрос в треде тупых вопросов. Если ты хочешь посраться и доказать что все неправы, а ты прав - создавай новый тред, а я не буду в нём писать.
117 664151
>>64149
Ты прав, судя по его посту >>64149 он не одупляет.
118 664152
>>64151
судя по его посту >>64150

self-fix
119 664153
>>64150

>это не звук и никуда он не распространяется за пределами волны


У тебя шизофазия?
мимо
120 664154
>>64151
>>64152
>>64153
Вместо того, чтобы спросить что непонятно, вы издаёте какой-то невнятный пук. Неконструктивно.
121 664156
>>64154
ОК, действительно что мы как школьники. Если ты считаешь, что в межзвездной среде с температурой 10 000K скорости звука нет, то вот тебе 3 вопроса:

1. Есть ли скорость звука в среде состоящей из холодного молекулярного водорода с плотностью 106 молекул на см3?

2. Есть ли скорость звука в среде состоящей из холодного молекулярного водорода с плотностью 103 молекул на см3?

3. Есть ли скорость звука в среде состоящей из холодного молекулярного водорода с плотностью 10 молекул на см3?
122 664159
>>64156

>Если ты считаешь, что в межзвездной среде с температурой 10 000K скорости звука нет


>10000К


>межзвездной среде


Саня, ты читаешь на что отвечаешь вообще, или просто на своей волне что-то отвлеченное говоришь?
123 664161
>>64156

>1. Есть ли скорость звука в среде состоящей из холодного молекулярного водорода с плотностью 106 молекул на см3?


Есть

>2. Есть ли скорость звука в среде состоящей из холодного молекулярного водорода с плотностью 103 молекул на см3?


Нет

>3. Есть ли скорость звука в среде состоящей из холодного молекулярного водорода с плотностью 10 молекул на см3?


Нет
124 664162
>>64156

>среде состоящей из холодного молекулярного водорода


В любых случаях число Кнудсена внутри молекулярных облаков соответствует свободномолекулярному движению. Это глубокий вакуум. В такой среде механическая передача звука в принципе невозможна. Можно вычислить условия образования ударной волны от летящего в такой среде предмета, и принять это за условную скорость звука. Но сам звук в такой среде невозможен. Даже турбулентные процессы, происходящие в такой среде, происходят на скоростях больше такой эквивалентной "скорости звука". хотя я не знаю - может и бывают какие-нибудь экзотические облака со сплошной средой? На грани гравитационного коллапса там, например.

>Если ты считаешь, что в межзвездной среде с температурой 10 000K скорости звука нет


Ещё раз, передача звука в плазме возможна, при определенных условиях (неравновесная плазма). Но это ионный звук, специфического рода явление со своими заёбами. В заряженных пылевых облаках. Опять же основано не на механическом взаимодействии.
125 664164
>>64161

>Нет


Как же так, среда есть - а скорости звука в ней нет?
>>64162

>это ионный звук


орнул
126 664167
>>64164

> Как же так, среда есть - а скорости звука в ней нет?


Это траленк тупостью уже пошёл?

> орнул


https://en.m.wikipedia.org/wiki/Ion_acoustic_wave
Просвещайся
1298572556471s.png71 Кб, 187x200
127 664171
>>64162

>это ионный звук

128 664172
>>64164

>Как же так, среда есть - а скорости звука в ней нет?


Это такая русская астрофизика, не удивляйся.
129 664182
>>64171
Ты специально проигнорил пост выше? Можешь проследовать нахуй. Это тред тупых вопросов, а не утверждений.
130 664183
>>64164

>Как же так, среда есть - а скорости звука в ней нет?


Есть среда - вакуум. Какова скорость звука в вакууме?
131 664239
В какую сторону вращается вселенная? По часовой или против?
132 664240
Сталкивались ли когда-нибудь 2 звёздные системы и 2 звёзды? Что происходило при этом?
133 664241
Если разогнать ядро атома до световой скорости, преодолеют ли электроны, вращающиеся вокруг ядра скорость света?
134 664243
>>64239
Она не вращается.
135 664244
>>64240
Да. Неясно.
136 664250
>>64241

>электроны, вращающиеся вокруг ядра


Товарищ Резерфорд, как вы можете писать на спейсач? Вы же мертвы
137 664252
1625419558319.jpg65 Кб, 750x413
138 664255
А дырку на мкс устранили? Ну в российском сегменте?
139 664275
>>64183

>Есть среда - вакуум


Ты толстый или глупый? Вакуум это когда нет частиц, а межзвездная среда и тем более молекулярные облака это десятки, а то и тысячи частиц на см3. Она полна вещества и в этой среде гуляют ударные волны, которые возникают при движении тел со сверхзвуковыми скоростями.
140 664278
>>64275

>Ты толстый или глупый? Вакуум это когда нет частиц, а межзвездная среда и тем более молекулярные облака это десятки, а то и тысячи частиц на см3. Она полна вещества и в этой среде гуляют ударные волны, которые возникают при движении тел со сверхзвуковыми скоростями.



Вакуум это когда длина свободного пробега больше размеров системы. А размеры системы определяются условием задачи.

Один и тот же межзвездный газ в задаче о охлаждении радиатора Вояджера - глубокий вакуум, в задаче о действии ударной волны от сверхновой - газ.
141 664279
>>64278

>Вакуум это когда длина свободного пробега


Это тебе голоса в голове сказали?
142 664280
>>64279
>>64275
Ебать ты поехавший конечно. С кем вообще ты борцуешь, тем более такими методами, в треде тупых вопросов? Кому ты вообще что пытаешься доказать, юродствуя? Может тебе стоит сходить на первый курс какой-нибудь шараги, вместо того чтобы надоедать анонимным учёным мирового уровня? Зачем ты оскорбляешь анона, который честно отвечает на твой тупой вопрос?
Это ведь тред тупых вопросов, а не тупых ответов или тупого односложного срача.
На, просвещайся, базовые определения.
https://ru.wikipedia.org/wiki/Число_Кнудсена
https://ru.wikipedia.org/wiki/Вакуум
Да, степень вакуума определяется числом Кнудсена. Представляешь? Больше единицы - вакуум, разреженная среда. Передача звука в ней невозможна. И кроме транзитных состояний, переход в свойствах качественный. Плазма - исключение, у неё при определенных условиях есть "боковой канал", по которому передача взаимодействия возможна, не механическим способом.
Всё, уходи и больше не приходи.

>>64275

>межзвездная среда и тем более молекулярные облака это десятки, а то и тысячи частиц на см3


Это глубокий вакуум, который не в каждой вакуумной камере получишь. Передача звука в такой среде невозможна без изменения её характеристик. Нету у него "скорости звука", еблан.

>в этой среде гуляют ударные волны


Долбоёб ты малолетний, ударные волны никуда не гуляют сами по себе, они рассеиваются очень быстро без воздействия. Их гонит тело, полностью изменяя среду в данном случае. А если речь не про молекулярные облака, а про ионизированную среду - в ней гуляет не звук, а ионно-акустические волны, взаимодействуют между собой заряженные частицы. Это отдельный феномен, который возможен в специфических случаях.
мимокрок

>>64278

>в задаче о действии ударной волны от сверхновой - газ


Эта среда не будет вести себя строго газодинамически вообще никогда.

>Вакуум это когда длина свободного пробега больше размеров системы. А размеры системы определяются условием задачи.


Когда длина свободного пробега больше расстояния между молекулами - газодинамика плотных сред заканчивается.
142 664280
>>64279
>>64275
Ебать ты поехавший конечно. С кем вообще ты борцуешь, тем более такими методами, в треде тупых вопросов? Кому ты вообще что пытаешься доказать, юродствуя? Может тебе стоит сходить на первый курс какой-нибудь шараги, вместо того чтобы надоедать анонимным учёным мирового уровня? Зачем ты оскорбляешь анона, который честно отвечает на твой тупой вопрос?
Это ведь тред тупых вопросов, а не тупых ответов или тупого односложного срача.
На, просвещайся, базовые определения.
https://ru.wikipedia.org/wiki/Число_Кнудсена
https://ru.wikipedia.org/wiki/Вакуум
Да, степень вакуума определяется числом Кнудсена. Представляешь? Больше единицы - вакуум, разреженная среда. Передача звука в ней невозможна. И кроме транзитных состояний, переход в свойствах качественный. Плазма - исключение, у неё при определенных условиях есть "боковой канал", по которому передача взаимодействия возможна, не механическим способом.
Всё, уходи и больше не приходи.

>>64275

>межзвездная среда и тем более молекулярные облака это десятки, а то и тысячи частиц на см3


Это глубокий вакуум, который не в каждой вакуумной камере получишь. Передача звука в такой среде невозможна без изменения её характеристик. Нету у него "скорости звука", еблан.

>в этой среде гуляют ударные волны


Долбоёб ты малолетний, ударные волны никуда не гуляют сами по себе, они рассеиваются очень быстро без воздействия. Их гонит тело, полностью изменяя среду в данном случае. А если речь не про молекулярные облака, а про ионизированную среду - в ней гуляет не звук, а ионно-акустические волны, взаимодействуют между собой заряженные частицы. Это отдельный феномен, который возможен в специфических случаях.
мимокрок

>>64278

>в задаче о действии ударной волны от сверхновой - газ


Эта среда не будет вести себя строго газодинамически вообще никогда.

>Вакуум это когда длина свободного пробега больше размеров системы. А размеры системы определяются условием задачи.


Когда длина свободного пробега больше расстояния между молекулами - газодинамика плотных сред заканчивается.
143 664281
>>64279
Вакуум это когда дышать нечем и кричать нельзя поэтому тихо.
144 664482
Аноны, из каких материалов делают сопла ЖРД (или РДТТ)?
На Falcon 9 сопла используются повторно? Если да, то что-либо известно про материал, из которого они сделаны?
145 664492
>>64280

>Эта среда не будет вести себя строго газодинамически вообще никогда.



А молекулярное облако холодного водорода с плотностью 103 молекул на см3 будет себя "вести строго газодинамически"?
В нем будут распространяться ударные волны?
Скорость звука при этом в среде будет равна 0 ?
Я правильно тебя понял?
146 664493
>>64482

>Аноны, из каких материалов делают сопла ЖРД


Медь?
sage 147 664499
>>64492
Ты заебал отвечать вопросами, свистни в хуй. Правильно-неправильно блять, ссышь что-то сказать самостоятельно.
148 664544
Поч спутников у планет открыли дохуя, а спутниц пока ни одной?
149 664546
>>64544

> Поч спутников у планет открыли дохуя, а спутниц пока ни одной?


это ошибка перевода, сам же понимаешь что какая нибудь луна точно девочка
150 664569
>>64544
у юпитера все спутницы и один пидор, например
151 664574
>>64499

>ссышь что-то сказать самостоятельно.


Я сказал, что в молекулярных облаках есть ударные волны, а следовательно есть скорость звука. Ты с этим не согласен? Ну так попробуй возразить.
152 664588
>>64250

> Товарищ Резерфорд


Твой одноклассник?
153 664589
>>64588
Нет, он умер задолго до того как я родился
154 664594
>>64250
а че он товарищ, это же англосакс был или кто он там по цвету
155 664597
>>64594
А тебе англосакс не товарищ?
156 664598
>>64597
ну да, я о них лучшего мнения
157 664606
Почему в космосе не используют сверхпроводники? Там же вакуум, если поместить его в тень то будет какраз достаточно низкая темпаратура. Или никто просто не нашел прикольного применения?
158 664607
>>64606

>если поместить его в тень


Например где?
159 664614
>>64606
Вакуум это отсутствие температуры, а не низкая. Их все равно нужно будет охлаждать, а с этим в космосе огромные проблемы.
160 664616
2 вопроса!
1) Чего больше во Вселенной: звёзд или планет?
2) Если скопление Шепли притягивает Аттрактор, который притягивает нас, то чем это всё закончится?
161 664617
>>64616

>1) Чего больше во Вселенной: звёзд или планет?


Видимо таки планет.

>2) Если скопление Шепли притягивает Аттрактор, который притягивает нас, то чем это всё закончится?


Тепловой смертью Вселенной.
162 664623
>>64617

>Тепловой смертью Вселенной.


А если на пару триллионов лет пораньше?
163 664624
>>64623
Не, вроде только локальная группа сольется.
164 664631
>>64606

>Там же вакуум, если поместить его в тень то будет какраз достаточно низкая темпаратура.



Температура среды на орбите Земли несколько тысяч градусов. Да, она разряженная, но понемногу эти частицы будут сталкиваться с твоим сверхпроводником и разогревать его. А так как в космосе передача тепла от тела возможна только излучением, а согласно закону Стефана-Больцмана эффективность передачи тепла излучением пропорциональна четвёртой степени его температуры. Так что твой сверхпроводник разогреется и перестанет сверхпроводить, либо придется устанавливать мощный радиатор для поддержания требуемой температуры.
165 664632
>>64614

>Вакуум это отсутствие температуры


Значит вакуума нет нигде, если твое утвержедение верно.
166 664635
>>64606
Используют криогенику изредка - это сложно и для этого нужен термодинамический баланс (радиаторы и/или зонтики). Например в космических телескопах юзают охлаждённые до криогенных температур сенсоры. Японские телескопы вроде самораспидорасившегося Astro-H/Hitomi имеют лютые трехступенчатые криокулеры, запиленные с какими-то ебовыми ноу-хау вроде баков из неонового льда. JWST ещё будет летать, прикрываясь огромным зонтиком, чтобы охлаждать сенсор.

А в сверхпроводниках просто особых задач нет, по-моему. Если бы были, то ReBCO вполне могли бы юзать, в теории. Может и юзают кстати, а я не знаю, но мне сомнительно.

Дауна >>64631 не слушай, контактная теплопередача пренебрежимо мала, отражать или компенсировать в первую очередь надо излучение Солнца и Земли (средняя температура что-то типа 20 градусов кажись), причём излучение Земли компенсировать даже сложнее, если говорить про криогенные температуры, т.к. энергия не размазана по всему спектру, а примерно совпадает со спектром излучения радиатора. Радиатор, который выведет теплопоток от них, без проблем на сдачу справится с контактной передачей тепла, энергии там несравнимые.
167 664636
>>64635

>Например в космических телескопах юзают охлаждённые до криогенных температур сенсоры.



Все верно. Если ты хочешь иметь низкие температуры своего аппарата в космосе то ты либо ставишь радиаторы с теплоносителем, как на МКС, либо охлаждаешь жидким азотом или даже гелием. Но когда гелий заканчивается с ним заканчивается время жизни твоего аппарата.
168 664637
>>64635

>излучение Солнца и Земли (средняя температура что-то типа 20 градусов кажись)



А что значит "средняя температура что-то типа 20 градусов"? Чего это температура?
169 664639
123
>>64616

>1) Чего больше во Вселенной: звёзд или планет?


Планет.

>2) Если скопление Шепли притягивает Аттрактор, который притягивает нас, то чем это всё закончится?


Для нашей местной группы галактик все эти аттракторы-хуякторы вообще похую. Расширение Вселенной уносит нас от этой залупы гораздо быстрее и сильнее, чем эта залупа притягивает нас. Мы с ней гравитационно не связаны, поэтому все закончится тем, что все эти пиздакторы улетят за горизонт видимой Вселенной и мы их никогда больше не увидим.
170 664648
>>64635
>>64636
А нельзя этот жидкий азот/гелий циркулировать в тепловых насосах, многоступенчатых, предположительно, чтобы он оставался и не заканчивался? Скажем, мой манянасос может создать градиент 100 градусов, ставлю три контура и на третьем у меня радиаторы сбрасывают 300 кельвинов?
171 664650
>>64648
Нужны просто компрессоры, чтобы сжимать азот обратно, ты же спрашивал про сверхпроводники, так вот с компрессорами у такой системы не будет энергоэффективности.
172 664651
>>64637
Средняя температура Земли. В тень от Земли можно спрятаться, но она всё равно служит теплообменником, её эффективная температура что-то уровня комнатной. На низких орбитах это актуально, где Земля приличный кусок неба занимает, а космос с его температурой в 4К - не 100%.
Температура Солнца же - 5800К, там такой поток идёт что мама не горюй, его сложнее отвести. (см солнечная постоянная - в окрестностях Земли прилетает 1366 ватт на квадратный метр плоского листа, если строго перпендикулярно к солнцу)

>>64636

>Но когда гелий заканчивается с ним заканчивается время жизни твоего аппарата.


Не обязательно, чего вдруг ему заканчиваться в закрытом цикле? Криокулер для того и нужен, чтобы обеспечить криогенные температуры в локальном объёме, как на JWST для MIRI. На Буране хотели ставить криокулер для кислорода, на телескопах сегодняшних ставят для гелия. Тащемта пассивное охлаждение работает до вполне криогенных температур и безо всяких криокулеров, вон и боинг и спейсикс всё рисовали заправочные баки с перманентным хранением водорода и метана в космосе (кислород боинг хочет хранить даже без зонтика, просто выкрасив бак правильной краской). Термодинамика вполне позволяет - принимаешь ты рассеянный спектр, а излучаешь в инфракрасном.
173 664652
>>64648
Можно, так и делают часто. Никаких сжиманий не надо. Это называется криокулер.
174 664654

>Нужны просто компрессоры, чтобы сжимать азот обратно


Уравнение Менделеева - Клапейрона говорит анону, что T= pV/R. Сжимая газ ты повышаешь его температуру, а тебе надо его охлаждать, а не надевать. Чтобы охладить газ нужно отвести от него тепло, в космосе это можно сделать только с помощью радиаторов, отвод тепловым излучением.
175 664655
176 664657
>>64651

>Не обязательно, чего вдруг ему заканчиваться в закрытом цикле


Потому что гелий нагревается и испаряется. Почитай историю Spitzer Space Telescope.
177 664659
>>64654
А ещё прочитай что такое термодинамический цикл. Скукоживаем - отводим образовавшееся тепло - раскукоживаем. Так-то первые ступени многих криокулеров действительно компрессионные, он об этом наверно.
178 664660
>>64659

>Скукоживаем - газ нагревается - отводим образовавшееся тепло - раскукоживаем.



Вот теперь верно.
179 664661
>>64657
И что? Сжижаем обратно криокулером. Причем тут спитцер - не понял, это далеко не единственный криогенный аппарат. У JWST тащемта газообразный гелий используется для следующего контура, и рециркулируется. У японских телескопов никаких потерь не было изначально, японцы были пионерами трехступенчатых криокулеров.
180 664663
>>64661

>И что?


Пойми простую вещь - чтобы сжижить газ нужно куда-то отвести от того газа тепло.
"криокулер" тут не поможет, запомни, есть 3 способа переноса тепла: теплопроводность, конвекция и излучение. В космосе возможен только третий способ переноса тепла - излучением. Эффективность этого способа описывается законом Стефана-Больцмана.
181 664668
>>64663
Ты говоришь прописные истины для старших классов с видом какого-то сокровенного знания. Ежу ясно что тепло надо отводить, зачем ты мне объясняешь что такое термодинамика КА? Я не задаю тут тупые вопросы, я на них отвечаю. Есть радиатор, он отводит тепло. Криокулер его генерирует, охлаждая при этом рабочее тело. Иди лучше объясни авторам этих аппаратов, что у них всё неправильно и бессрочное хранение криогеники или рециркуляция невозможны.
182 664669
>>64660
А, ну раз аноним одобрил, спасибо. Я-то не знал.
183 664670
>>64668

> Иди лучше объясни авторам этих аппаратов, что у них всё неправильно и бессрочное хранение криогеники или рециркуляция невозможны.


А почему таки JWST не регенерирует хладагент? Гелий утекает куда-то что ли испарившись?

мимо дебил
184 664675
Какие светильники на МКС используются?
Раньше там лампы накаливания стояли или дневного света или сразу диоды?
185 664679
>>64670
Гелий в нём и есть хладагент. Просто газообразный. Почему не жидкий - у JWST целевая температура инструмента 6.2К (там нестандартный полупроводниковый детектор, который только при ней и работает), и охлаждают до неё. Жидкий гелий это лишняя ебля - он сжижается при ещё меньшей температуре (4 с небольшим кельвина), сверхтекучий, и будет менять оптические свойства детектора.
186 664681
>>64675
Флуоресцентные всегда юзали и юзают.
187 664682
>>64668

>Есть радиатор, он отводит тепло.


ОК, тогда вопрос для >>64668

> старших классов


У тебя есть 10 кг газообразного азота при T=300К. Ты хочешь получить из него 10 кг жидкого азота с T=70К. Определи количество теплоты, отданное газом.
188 664683
>>64681
А если они разобьются, там же газы, стекло, это все?
189 664684
>>64679
Спасибо за уточнение. Но я все равно не пони почему он закончится?
190 664686
>>64684
Он не закончится. Кто тебе это сказал? Наркоман, у которого криокулеры не работают? Так не слушай наркоманов, анонимные учёные мирового уровня иногда злоупотребляют веществами, бывает.
Срок жизни JWST ограничен топливом на борту, а не запасами холода. Для холода у него зонтик есть, который его от солнца закрывает, криокулер, и излучатель, который сбрасывает тепло в окружающее пространство.
191 664687
>>64679

>Гелий в нём и есть хладагент. Просто газообразный



А ты уверен, что понимаешь значение слова "хладагент"?

>температура инструмента 6.2К (там нестандартный полупроводниковый детектор, который только при ней и работает), и охлаждают до неё



Газообразным гелием?
image.png488 Кб, 631x600
192 664688
>>64686

>Срок жизни JWST ограничен топливом на борту, а не запасами холода. Для холода у него зонтик есть, который его от солнца закрывает, криокулер, и излучатель, который сбрасывает тепло в окружающее пространство.


Спасибо, теперь стало понятно. Недопонимание у меня возникло из-за того что я подумал что срок ограничен охлаждением а не топливом.
193 664689
>>64686

>Он не закончится.


А как ты представляешь себе процесс когда гелий с температурой 300K забирает тепло у инструмента с температурой 6K?
Любопытно послушать.
194 664694
>>64687

>А ты уверен, что понимаешь значение слова "хладагент"?


Теплоноситель, если ты любишь бессмысленно доёбываться.

>Газообразным гелием?


Представляешь, да? На JWST нет жидкого гелия, равно как и температур для него. Перестань курить то что ты куришь.

>>64689

>гелий с температурой 300K забирает тепло у инструмента с температурой 6K


Ой, да просто сходи нахуй, идиот. Я не знаю что происходит в твоей голове, но тебе точно надо остудить мозг, судя по чуши которую ты несёшь. Не отвечай мне.
195 664695
>>64694
Ты чего такой дерганный? Я лишь спросил тебя - как ты себе представляешь процесс при котором теплый газ забирает тепло у холодного тела. Это просто вопрос, бро.
nebrat.mp4278 Кб, mp4,
640x360, 0:04
196 664701
>>64695
Это ты себе такой процесс представляешь, в своей голове.
197 664724
>>64701
Ты чего такой обидчивый? Я не оскорбляю тебя, я спросил как ты себе это представляешь?
198 664739
>>64686
А когда топливо кончится, он же еще сможет поработать крутясь на гиродинах пока не слетит с Л2 в ебеня?
199 664745
Возвращаясь к вопросу о гравитации vs центробежного ускорения
В теории расширяющейся земли на вики перечислены 4 варианта причин:
https://en.wikipedia.org/wiki/Expanding_Earth
1.1Expansion with constant mass
1.2Mass addition
1.3Decrease of the gravitational constant
1.4Formation from a gas giant

Как насчёт такого 1.5 варианта что земля распухла из-за возросшего в следствии космической катастрофы ускорения?

Может кто-нибудь примерно почувствовать насколько быстрее должна раскрутится земля что бы её раздуло в два раза, при том что масса примерно та же?

Если допустить что земля например не крутилась вообще, а потом её крутануло до нынешнего состояния, наверное прирост размера всё равно будет гораздо меньше чем на пикрил?
200 664757
>>62981 (OP)
На этой вашей МКС ганджу редиску выращивают под непрерывным светом или соблюдают режим день-ночь?
201 664764
>>64757
соблюдают
202 664771
Что такое сила радиационного трения и откуда она берется?
203 664782
>>64745
Да ладно. Континенты не разъехались и промежутки потом водой залились. Есть такая вещь как континентальные плиты, они все друг друга касаются. Их низкие части залиты водой, верхние нет. Все просто же, какая еще расширяющаяся земля?
204 664789
>>64771

> Что такое сила радиационного трения и откуда она берется?


Из головы шизофреников, которые пишут новые религиозные догмы навуки
205 664791
Ну вот, похоже все нормально теперь, можно будет с большой вероятностью ждать запуска телескопа Джеймса Уэбба, однако не пойму каким богам молиться чтобы снова запуск не перенесли?
image.png804 Кб, 862x485
206 664793
>>64791

>каким богам молиться чтобы снова запуск не перенесли?


Одину или пикрилу.
207 664794
>>64771
Радиационное трение, реакция излучения, лучистое трение, торможение излучением — сила, действующая на заряженную точечную частицу (например, электрон), со стороны её собственного электромагнитного излучения, вызываемого неравномерностью движения этой частицы.

Если тебе нужно представить это физически, то как зримая модель подойдут аномалии ускорения Вояджеров.
208 664813
>>64794

>лучистое трение


Звучит как название фильма от Brazzers
209 664815
>>64794

>сила, действующая на заряженную точечную частицу (например, электрон), со стороны её собственного электромагнитного излучения



А разве не нужно второе тело, которое эту силу создает? Откуда сила то берется?
210 664817
Если открыть порталы в пустыне, где +50оС и в антарктике с -50оС, какой воздух пойдет через порталы? Теплый воздух из пустыни или холодный?
211 664818
>>64817
Теплый из пустыни.
212 664826
>>64817
Откуда ветер подует, оттуда и пойдёт. Меняться будет.
213 664827
>>64826
>>64818
>>64817
Т.к. полюса ниже экватора (они ближе к центру земли) и там не действует центробежная сила, следовательно давление воздуха там должно быть в целом выше. Отсюда вывод что ветер должен дуть с полюсов.
Но я дебил который забыл про всех этих больцманов и не знаю как температура воздуха с этим кореллирует.
214 664828
>>64817
Кстати, вместо того чтобы возводить число в степень "о", ты можешь нажать Win+. и открыть допклаву где можно выбрать такие символы как ° и ℃. Еще там уебищные смайлики которые никак не отключить, к сожалению. Хотелось бы чтобы по дефолту открывались именно полезные символы а не это говно, но майкрософт не читает мои письма.
215 664829
Почему так дрочат на такого крутого астронома Карла Сагана, если Карл Саган не был крутым астрономом? Он много пиздел, да, разводил пустую псевдофилософию и многословно излагал очевиднейшие "теории" (типа жизни на Юпитере, которая - о, откровение! - могла бы летать в атмосфере на пузырях с водородом). К тому же чсв-шным мудаком был Карл Саган по типу Джобса. Откуда хайп? Где достижения? Негр Тайсон такой же балабол, чому и негра превозносят-уважают-уважают? Кто эти люди вообще, авторитеты для хипстерков и быдла, так чтоли?
image.png569 Кб, 3531x593
216 664830
>>64829
Они буквально учёные, доктора наук со множественными публикациям, ты чего?
И ты так говоришь, будто популяризация науки это что-то плохое.
007.jpg33 Кб, 525x477
217 664831
>>64827
смотри чего нашёл
image.png287 Кб, 512x384
218 664833
>>64831
Глабодарю
219 664834
>>64829
Забавно это читать от двачера, который наверняка даже бакалавриат хоть какой-нибудь не закончил. Не говоря уже о кандидатской/докторской степени
220 664836
>>64830

>чето там предположил (на Венере жарко)


>в чем-то там участвовал (уровня минорных попизделок с большой командой разработчиков Маринера)


>годами "разрабатывал теорию" что на Юпитере летают медузы-пузыри и их едят рыбы-пузыри


>АЙЯЙ КАРАУЛ ПОТЕПЛЕНИЕ! СКОРО БУДЕМ ВТОРОЙ ВЕНЕРОЙ(ой дебииил!)


>оче много пиздел, пиарился, чсвшничал как голимая хипстота, норм учёным надоело, выпёрли его из Академии


>возможно (нету ссылки на статью, значит нет и статьи) придумал что на Титане жидкий метан



Ну охуеть достижения, вот это светоч!
221 664841
>>64836
А что, эти публикации не научные публикации, он из-за того что ты не видишь ценность в этих работах теперь не доктор наук? Top Secret в ВВС и Secret в НАСА ему по блату выдали?
А что по-твоему настоящий светоч, хотел бы знать?
И ты не ответил, что плохого в популяризации науки.
222 664842
>>64834
Кандидатов/PhD полно, алё! И никто их не возводит в культовый статус, всех и каждого. Вот если он не просто защитил дисер, но и много сделал, вот это крутой учёный. А не "много пиздел на публику, на уровне школьного учителя по физике".
И да, этот твой очене крутой "сначала добейся"-довод как бы это сказать, ну...не работает лууул
223 664844
>>64842

>И никто их не возводит в культовый статус, всех и каждого.


Верно. Ты не пробовал задумываться, почему так? Когда ты свои вопросы задавал, у тебя не было даже намёка на идею в чем причина такой популярности? Или ты просто как типичный бунтарский подросток-контрариан решил излить желчь на что-то популярное?
224 664845
>>64841
Светоч бы много смотрел в телескоп, писал всякие новые формулы, находил закономерности, годами проводил наблюдения чтобы потом понять то, чего до него никто не знал. Анализ неочевидных скрытых взаимосвязей, смелые гипотезы, теоретическая работа, разработка методов, эксперименты. А не "Ооо я вам щас расскажу че, на Юпитере рыбы-пузыри, а на Венере жарко, прикинь? И на Марсе нету растительности. А ещё я посветил ультрафиолетом и там появились аминокислоты. Это не эксперимент Миллера-Юри, повторяю, не эксперимент Миллера-Юри!"
225 664846
>>64844
Не люблю следовать стадному мнению. Люди обычно мало сведущи в астрономии, а я достаточно, чтобы испытывать недоумение от "достижений" этого заурядного болтуна в микрофон. Как популяризатор - ок, один из, как "великий астроном" - нет, не великий нисколько, не надо делать из меня идиота.
226 664847
>>64845

>Светоч бы много смотрел в телескоп


Крайне глупо во второй половине 20 века.

>писал всякие новые формулы


Всякие новые формулы и пишут. Открывай публикации и смотри.

>находил закономерности


Саган и находил, блжад.

>годами проводил наблюдения чтобы потом понять то, чего до него никто не знал.


А если не годами а быстро, то не считается?

>Анализ неочевидных скрытых взаимосвязей, смелые гипотезы, теоретическая работа, разработка методов, эксперименты.


У Сагана как раз и было, лол. Нигрограсс вот не особо тут отличился.

>А не "Ооо я вам щас расскажу че, на Юпитере рыбы-пузыри, а на Венере жарко, прикинь?


А почему нет?
Еще раз: что плохого в популяризации науки?

>>64846

>Не люблю следовать стадному мнению.


Ага. Я полагаю, что тебе 20±3 примерно, это совершенно обычное подростковое бунтарство - отрицать что-то популярное просто по причине популярности. Может если ты действительно такой умный каким себя считаешь, увидишь, что многие популярные вещи популярны неспроста и ненависти не заслуживают и показательно ненавидеть что-то популярное не делает тебя особенным. Иронично, это тебя как раз делает как и большинство твоих ровесников.

>Люди обычно мало сведущи в астрономии, а я достаточно, чтобы испытывать недоумение от "достижений" этого заурядного болтуна в микрофон. Как популяризатор - ок, один из, как "великий астроном" - нет, не великий нисколько, не надо делать из меня идиота.


А кто и где тебе говорит что он великий астроном? Саган и Ниграсс известны в первую очередь как популяризаторы науки.
227 664854
Откуда инфа что Сурдин пассивный гомосексуалист?
228 664867
https://www.youtube.com/watch?v=OR5PpfkgXr4
Почему этот чувак говорит, что тепловой смерти Вселенной не случится?
229 664868
>>64867
Вот фиг знает. Для этого, наверное, надо видео посмотреть, но этого я делать не собираюсь.
230 664869
А хули астероиды ещё не разъебали нашу планету, почему она не притягивает их своей гравитацией?
231 664871
>>64869
Планете-то похую на астероиды. Ну уебался и хуй с ним, магменная дырка заживет через тысячонку лет.

> почему она не притягивает их своей гравитацией?


Притягивает.
232 664873
Всё, я конкретно заебался, у вас есть видосик про гравитацию в космосе на русском для полных дебилов, ну вот ни в какую я не понимаю, почему во вселенной нихуя не меняется и всё летает на своих орбитах.
Почему Земляшка не притягивает все астероиды, почему Солнце не притягивает Земляшку, почему само Солнце не притягивает ещё большая звезда и почему 1 галактику не притягивает к другой, более большой, галактике
233 664875
>>64873

>Почему Земляшка не притягивает все астероиды, почему Солнце не притягивает Земляшку, почему само Солнце не притягивает ещё большая звезда и почему 1 галактику не притягивает к другой, более большой, галактике


Но анончик... Притягивают же. Все орбиты из-за этих притяжений и меняются. Просто они... орбиты. Луна не упадет на землю от того что моя мамаша родилась. Рождение моей мамаши - это уникальное необъяснимое событие с точки зрения науки, она весит как еще одна земля. В итоге луна стала крутиться по вытянутой орбите с момента появления моей мамаши.

Я думаю тебе поможет поиграться в Universe Sandbox (рекомендую), Kerbal Space Program или накодить свою простенькую визуализацию гравитации (крайне рекомендую)
orbit.jpg59 Кб, 849x719
234 664880
>>64873

>Почему Земляшка не притягивает все астероиды, почему Солнце не притягивает Земляшку, почему само Солнце не притягивает ещё большая звезда и почему 1 галактику не притягивает к другой, более большой, галактике


Притягивает. Ты вероятно имеешь в виду, почему Луна не падает на Землю, Земля - на Солнце, и так далее? Ну смотри, пикрелейтед. Это планета, на планете высокая башня, с башни ты бросаешь камень вбок, сначала несильно (чёрная траектория), потом всё сильнее (красная и синяя). Они падают на землю. Наконец, если бросить камень с достаточной скоростью, то он будет падать, но промахиваться мимо земли, описывая окружность. Слева нарисованы два положения камня на этой окружности и направления притяжения камня землёй. Земля притягивает камень постоянно к своему центру, создавая ускорение, которое в данном случае будет постоянно направлено перпендикулярно направлению полёта.

Да, видео наверняка полно по этой теме, ищи по запросу "орбитальное движение".
235 664881
>>64880
Добра тебе за терпеливое объяснение начал орбимеха

мимо другой
236 664882
>>64873
1. Меняется. Луна например отдаляется от Земли потихоньку, при образовании она была в разы ближе. Кометы падают на Солнце. Дохуя чего меняется.

2. Всё летает на своих орбитах потому, что ещё при формировании Солнечной системы всё что могло посталкиваться или быть выкинутым нахуй - посталкивалось и было выкинуто нахуй. Осталось только то, что удачно расположилось по орбитам.

3. Всё притягивает друг друга, и Земля например постоянно падает на Солнце, но всё время промахивается, потому что летит вбок с удачной скоростью.

А так, никто даже не знает на 100%, стабильна ли Солнечная система - например есть небольшой шанс, что через 12 миллионов лет Меркурий дестабилизируется и улетит нахуй, потянув за собой всё остальное.
237 664883
Объясните смысл недавнего открытия галактики, существующей 12,4 млрд лет. Это же не значит, что мы дошли почти что к краю вселенной? Слишком абсурдно, но тогда что это означает?
238 664885
>>64854
Удвою анона. Манера речи располагает к таким выводам, но пруф?
И кто его партнёр?
239 664889
>>64885
Физик Сергей Попов
давно известно это
240 664892
>>64883
Мы никуда не "шли", мы просто видим тот свет, который до нас долетает. Поскольку вселенная существует ~13,7 миллиардов лет, то соответственно именно свет, испущенный примерно столько времени назад, приходит от объектов на краю наблюдаемой вселенной. Именно наблюдаемой вселенной - то есть это такой пузырь вселенной, от объектов внутри которого до нас успел долететь свет, а от более далёких объектов (если они есть), свет до нас не долетел и никогда не долетит. Размеры вселенной за пределами наблюдаемой вселенной мы не знаем.

Вообще, наша галактика тоже примерно 12 миллиардов лет существует, просто от тех галактик мы сейчас видим свет, испущенный тогда, на раннем этапе формирования галактик.
241 664901
>>64892
Если Земля существует 4 милларда лет, свет той галактики шёл 12,5 млрд. лет, а вселенная существует 13 млрд, то не должен был бы свет долететь до нас быстрее?
Пиздец у меня в голове муть, пойду полежу
242 664906
То есть газовые облака, из которых образуются звёзды, конечны и больше ни при каких обстоятельствах не появятся вновь?
243 664907
>>64906
Появятся при взрыве звезд и прочих подобных событиях
244 664909
У меня есть тупой вопрос.
Доказывает ли эксперимент квантового ластика с отложенным выбором, что мы живем в матрице?
Как вообще можно объяснить результаты этого эксперимента?
245 664913
>>64901
Не очень понял вопрос, при чём здесь возраст земли? Вот смотри, вселенная появилась грубо говоря 13 млрд лет назад, через полмиллиарда лет сформировались разные галактики в разных местах расширяющейся вселенной и начали испускать свет. От тех галактик, которые к нам поближе, первый свет дошёл быстро и после этого отсюда можно было наблюдать, что там происходит, как они развиваются. От более далёких свет пришёл позже, а от самых далёких (из тех, что мы видим) - вот только сейчас.
246 664916
>>64913
Да я покушал, полежал и тоже догнал в чём дело, теперь стыдно...
247 664918
>>64913

>вселенная появилась грубо говоря 13 млрд лет назад



Какого хрена мы тогда наблюдаем вблизи горизонта видимости массивные объекты типа галактик, ведь тогда они еще не могли сформироваться?

мимодругойанон
248 664922
>>64918
Могли, с -13,7 миллиардов лет до -12,5 миллиардов лет почти миллиард лет прошёл.
https://en.wikipedia.org/wiki/Chronology_of_the_universe#The_Dark_Ages_and_large-scale_structure_emergence
249 664924
>>64909
ну матрица это классический чайник по определению, так что её ни доказать ни опровергнуть
а по вопросу ты как то более развёрнуто задай, не пони
dfrcgyieQQxEJ4eZXqgLSY-1024-80.jpg44 Кб, 650x635
250 664930
251 664931
Почему на всех фотачках галактик по центру настолько охуенно светло, что там будто бы одна огромная звезда?
Это там просто самое плотное скопление звёзд?
252 664936
Я так понимаю сейчас есть 3 самых вероятных конца Вселенной:
1) Тепловая, когда Вселенная будет бесконечно расширятся, но никаких новых процессов в ней происходит не будет, звёзды будут затухать, остатки засосут чёрные дыры, которые в дальнейшем тоже исчезнут и всё... конец...
2) Большой Разрыв, где скорость расширения нагнёт гравитацию и расщепит всю Вселенную нахуй
3) Ситуация, где гравитация нагнёт скорость расширения и скомкает всю Вселенную в новую сингулярность
Я всё верно понял? Ничего не забыл?
253 664937
>>64936
Скорее это не отдельные возможные сценарии конца вселенной, а его этапы.
254 664938
>>64936
Скорее всего конца не будет, Вселенная будет вечно расширяться.
255 664939
>>64936

> Я так понимаю сейчас есть 3 самых вероятных конца Вселенной:


> 1) Тепловая, когда Вселенная будет бесконечно расширятся, но никаких новых процессов в ней происходит не будет, звёзды будут затухать, остатки засосут чёрные дыры, которые в дальнейшем тоже исчезнут и всё... конец...


> 2) Большой Разрыв, где скорость расширения нагнёт гравитацию и расщепит всю Вселенную нахуй


> 3) Ситуация, где гравитация нагнёт скорость расширения и скомкает всю Вселенную в новую сингулярность


> Я всё верно понял? Ничего не забыл?


Ты, блядь, 50 лет еще проживешь, но тебя ебет что когда-то там через охуиллиард лет произойдет.
256 664941
>>64936
Большой разрыв - наиболее маловероятный сценарий. Скорее всего будет тепловая смерть.
257 664946
>>64936
Большой разрыв случится. Вселенная с ускорением самораспидорашивается, это факт.
258 664947
>>64946
Так называемый большой разрыв, также как и технологическая сингулярность - абсолютно пустые, надуманные "термины" которые кто-то когда-то спизданул, ученый уровня "мегаастронома" Карла или купившего отжавшего диплом нигры Тайсона. Так вот, это самое расширение вселеной действует только на намтолько огромные куски, которые уже не связаны грааитационно. А те куски, что грпвитационо связаны, остаются на месте. Самый большой такой кусок это скопление галактик, в нём от десятков до сотен (сотен, астроном Карл!) галактик. Всё это останется навсегда, никуда не улетит, мы не улетим, те кто будут после нас не улетят, черноты не будет, звёзды с неба не исчезнут и твой палец на атомы с каких-то херов не распадётся. Понятно, анон?
https://ru.m.wikipedia.org/wiki/Скопление_галактик
259 664948
>>64947
Хватит троллить тупостью, в /b/ вернись и там воюй с наукой и здравым смыслом.
260 664949
>>64948
По сути он прав, не звезди мне тут. Большой разрыв жопы - это маргинальная гипотеза, ничем не подтвержденная, и даже скорее не гипотеза, а просто научпокерский мемас, с которым брейнлеты вроде тебя носятся по интернетам, как с писаной торбой.

И да, современную "астрономию" было бы неплохо кансельнуть до кучи, ибо довничей типа Тайсона, по-рейдерски захвативших отрасль, там и так слишком много дохуя. Говна не надо.
sage 261 664950
Опять просветлённые борцуны с ветряными мельницами на дваче, знающие СТРАШНУЮ ПРАВДУ. Ну что ты будешь делать.
262 664951
>>64949
Большой разрыв - совершенно логично выводится из НАБЛЮДАЕМОГО ускорения расширения вселенной. Разрыв ТОЧНО случится, это ОЧЕВИДНО, кретин ебаный. Съеби отсюда.
263 664954
>>64951

>Справедливость этой гипотезы сильно зависит от природы тёмной энергии, а именно, от параметра w, равного отношению давления тёмной энергии к её плотности (см. уравнение состояния). Если w < −1, то Вселенная будет ускоренно расширяться, и величина масштабного фактора станет равной бесконечности за конечное время.



Вот когда докажут, что w < −1, тогда и поговорим. В любых теоретически мотивированных моделях w больше либо равно −1.
ты ОБОСРАЛСЯ.
264 664955
>>64951
Второй закон термодинамики сообщает что энтропия - мера неупорядоченности - со временем растёт. Совершенно очевидно и логично, что если энтропия увеличивается в направлении будущего, то, если смотреть в прошлое, она должна уменьшаться и когда-то в прошлом быть очень низкой. Следовательно, Большой взрыв должен быть очень высокоорганизованным процессом, с очень малым элементом энтропии. Согласен?
265 664958
>>64948
А может ты нахуй идешь? Да, пожалуй что так, FOCK U, MUDACHEQUE! Пососи мой stellar-бибу лол кек пук взрыв хлопок
266 664968
>>64144

>Либо размотать за собой бобину тонкого стального троса, километров на 20 - и держаться за него. Чтобы он тебя не догнал, надо вовремя отцепиться.


Не понял. А как это должно работать?
267 664980
Можно ли укоряться, но при этом положение в пространстве измениться не будет?
268 664981
>>64980

> Можно ли укоряться, но при этом положение в пространстве измениться не будет?


Таблетки
269 664982
>>64981
А без таблеток? Вот ты ускоряешься на 9,8 м/c2 но ты же как сидел сейчас на пропуканом диване так и сидишь. А если бы на тебя не действовало ускорение 9,8 м/c2 что бы с тобой было? Улетел бы.
270 664984
>>64982

> А без таблеток? Вот ты ускоряешься на 9,8 м/c2 но ты же как сидел сейчас на пропуканом диване так и сидишь. А если бы на тебя не действовало ускорение 9,8 м/c2 что бы с тобой было? Улетел бы.


Две смирительные рубашки этому господину
271 664993
Джеймс Уэбб будет подвозить наиболее качественные изображения экзопланет при их прямом наблюдении?
272 664998
>>64993
Вероятно что такой телескоп позволит наблюдать большее количество экзопланет прямым методом, чем было доступно ранее, но не более того. Если под качеством ты понимаешь количество пикселей, и то что будут видны какие-то детали на поверхности планет, то лучше явно не будет. Как бы по разрешающей способности Джеймс Уэбб довольно посредственный.
273 664999
>>64993
Это инфракрасный телескоп, по большей части, и через коронограф можно будет наблюдать просто другую картинку, у него свои задачи. Оптическим же подвезли адаптивную оптику за прошлые пару десятилетий, позволяющую практически игнорировать атмосферу - поэтому большего разрешения чем на наземных одноапертурных VLT/GTC/Keck/Subaru не выйдет, а уж тем более в скором будущем входящими в строй Большим Магеллановым телескопом или ELT с их гигантскими зеркалами.
274 665006
>>64999

>большего разрешения


>Большим Магеллановым телескопом или ELT с их гигантскими зеркалами.


>одноапертурных


Так вот если речь об именно "одноапертурных", то есть не каких-то там интерферометрах, то каким образом у тебя будет хоть какое-то "разрешение" даже в таком случае? Ведь диаметр зеркал у существующих телескопов максимум около 10 метров. Ну ладно, можно ожидать что в обозримом будущем появятся под 30 метров.

Как бы для разрешающая способность телескопа в видимом диапазоне равна 140/D угловых секунд, где D - диаметр объектива в мм. Разрешающая способность 30 м телескопа будет около 0,005 угловых секунд. Однако на одну линию разрешения должно приходиться несколько пикселей, допустим 3-4 (многим же привычнее в пикселах оценивать). Допустим экзопланета будет размером с Юпитер. При наблюдении с Земли, Юпитер имеет угловой размер около 50 угловых секунд (при расстоянии около 5 а.е. до него). Соответственно можно получать изображение Юпитера с размером 40к на 40к пикселей, что конечно здорово. Но вот до ближайших экзопланет 300 тыс. а.е., соответственно видимый размер экзо-Юпитера на таком расстоянии будет в 60 тыс. раз меньше, чем видно с Земли. Так что о какой-то детальности (с "одноапертурными" телескопами) и речи быть не может.
275 665009
>>65006

>Так вот если речь об именно "одноапертурных", то есть не каких-то там интерферометрах, то каким образом у тебя будет хоть какое-то "разрешение" даже в таком случае?


Никакое, пиксели на планету. А ты ожидал что будет БКП на внесолнечном Юпитере видно? Все напрямую разрешённые на данный момент экзопланеты выглядят как пикрил, скажи спасибо что там больше одного пикселя, и то это хорошо освещённые газовые гиганты всякие.
Интерферометры больше для звёзд подходят, для экзопланет чувствительность у них не оче. Синтетическая апертура большая, но отношение к собирающей поверхности падает с её ростом.
276 665010
>>65009
пикрил-то я и забыл
277 665016
>>65009

>Никакое, пиксели на планету.


Чтобы хоть что-то там увидеть, нужно хотя бы три-четыре хороших таких пикселя (ну или точнее один элемент разрешения). А для 30 м телескопа будет 0,7 пикселя для ближайшего экзо-Юпитера, как я писал чуть выше.
Так что с хрена ли это вдруг возникнет какое-то разрешение (по поверхности самих планет)?
278 665019
>>64998
Возможность более подробного изучения спектра.

Благодаря расчётам учёные смогли прикинуть примерный вид Плутона ещё до того, как мимо него пролетел аппарат новые горизонты. Примерное такое же хотелось бы и с экозпланетами.

Или открытие экзолун.
279 665021
>>65009
В смысле падает: разнесли 4 40-метровых в интерферометр с плечом в 200м. Сколько света соберут, неужто не как 4×40м?
1.jpg154 Кб, 1000x667
280 665065
Почему ее просто нельзя обшить какой-нибудь хуйней, которая поглощает радиацию?
281 665067
>>65065
А зачем? Там и так норм.
282 665068
>>65065
есть ли в этом хоть какой-то смысл?
283 665069
>>64867
потому что...
284 665071
>>65067
>>65068
А хули они тогда больше года не работают на станции, если там все норм и не фонит?
285 665072
>>65071

> не работают


работают
286 665073
>>65071

>А хули они тогда больше года не работают на станции


>работают


Мы не работаем, а капчуем.
мимо Дубров
287 665077
>>64854
Хз. Могу лишь сказать, что Сурдин педераст по той части, где про Венеру рассказывал у шихман. Если уж взялся рассказывать, то рассказывал бы до конца. А не байтил
288 665078
>>65065
Потому что радиация это не единственный фактор влияния. И это дорого, сложно и нет смысла.
>>65071
Потому что радиация это не единственный фактор влияния и это дорого, сложно и нет смысла.

Если интересно,начни с этого https://habr.com/ru/post/362861/
Потом можешь почитать чего-нибудь из научных трудов (на русском языке валом, благо первого хомо в космос запустили)

Там помимо радиации приколов хватает, и хоть большинство этих последствий можно кхм, задавить, не всё так просто и однозначно. Если проще сказать: мы для микрогравитации не очень-то годимся на фулл-тайм.
16258579234390.jpg51 Кб, 692x706
289 665082
>>65016
Не понял, ты сомневаешься что планеты вообще в принципе можно разрешить? Так выше же есть фотки.
Композиты, суперразрешение, локальная оптимизация и т.п. Как я это понимаю, диффракционный предел это очень грубая оценка, под него можно залезть разными способами, с оговорками.
Пик 1 >>65010 выглядит вот так, не знаю сколько там дисков Эйри грубо говоря вмещается, но это определенно не точечный источник. У больших телескопов будет больше пикселей, вот и всё разрешение.
.png4 Кб, 420x420
290 665083
>>65082
Блять. Вот так он выглядит.
291 665088
>>65021

>Q: What kind of advantages will the ELT offer, compared with the VLTI interferometer, which, by combining light from various individual telescopes, makes up a larger virtual telescope than the ELT?


>A: By combining four telescopes using interferometry, the achieved resolution is the same as a single telescope whose diameter is equal to the maximum distance between the individual telescopes in the interferometer. In this way, the VLTI allows us to create a virtual telescope up to 130 metres in diameter when combining the 8.2-metre Unit Telescopes (UTs), or up to 200 metres in diameter when combining the 1.8-metre Auxiliary Telescopes (ATs). In spite of this, the 39-metre ELT will have considerable advantages with respect to the VLTI. First, the ELT will have a collecting area that is a hundred times larger than the four VLT ATs combined. This means that the ELT will collect one hundred times more light and therefore will allow observations of much fainter objects. Secondly, in order to create images from a virtual telescope, the VLTI has to observe over a long period of time. Thinking of the four 1.8-metre ATs as a tiny fraction of the full 200-metre virtual telescope, the gaps in the 200-metre virtual mirror must be "filled in" by moving the ATs between positions on the mountaintop, or allowing the Earth's rotation to reorient them relative to the astronomical target. For the ELT the "aperture" is fully filled at all times. Last, but not least, the data from the VLTI must go through a very complex form of data processing. This method only provides a very tiny field of view and currently only works well on relatively bright objects. In conclusion, the VLTI is very powerful for certain applications, but can only compete with the ELT on a fraction of science projects.

292 665161
>>65083
Что ты пытаешься доказать невнятной мешаниной из пикселов, у которой ты сам не знаешь какой масштаб?

>>65088
Только правильнее объяснить так: уменьшается не общее количество света, а количество, приходящееся на отдельный пиксель. Но планеты обычно довольно яркие. Если что, то поверхностная яркость не зависит от расстояния, конечно если по пути свет не слишком существенно ослабляется газом/пылью. Вот Луна у нас яркая? По моему весьма яркая. Если фотографировать обычным фотоаппаратом, то с какой минимальной диафрагмой будет приемлемая экспозиция?
293 665174
Есть у кого видео МУСОРА пролетевшего рядом с МКС? Почему все СМИ замолкли и нет ни одного видоса? По рентв на каждый пук из соседней квартиры по звонку рендом бабки репортируют, а тут чуть МЕЖДУНАРОДНУЮ станцию не разбомбило, а все ебальники захлопнули и молчат.
294 665175
>>65174
Потому что ничего особенного не произошло, просто СМИ решили раздуть как всегда из ничего сенсацию. Ну какой-то кусок мусора пролетел в 4 км, это регулярно происходит.
295 665177
>>65174
Потому что не разбомбило, говно постоянно рядом крутится и это не новость.
296 665183
>>65175
>>65177
Ну какое никакое событие, хоть бы сфоткали. Они один фиг всякую парашу постоянно снимают. А может власти замалчивают, а?
297 665184
>>65183
Они постоянно мусор снимают. И сами же и выбрасывают во время ВКД. Это буквально неновость, всем похую.
298 665199
>>65161

>Что ты пытаешься доказать невнятной мешаниной из пикселов, у которой ты сам не знаешь какой масштаб?


Доказать?.. Наверно ничего. Я просто не понимаю с чем ты споришь и зачем. Ты пытаешься сказать что экзопланеты до сих пор ещё не разрешили как не-точечный источник? Разрешили же, это ведь явно не точечный - независимо от пиксельного масштаба. Надо будет разобраться достоверно (лень ковырять журналы), но по-моему даже эта пикча явно показывает что планету видно больше чем точка, не?
299 665201
>>65174
Видео? Ты представляешь какие там относительные скорости? Километры в секунду. Даже определить что в станцию попала пиздюлина, это целая проблема - для этого ебовые технологии сочиняют, типа волоконно-оптических датчиков натяжения https://technology.nasa.gov/patent/LAR-TOPS-245 , интерферометров, микрофонов, хуёметров.

>По рентв на каждый пук из соседней квартиры по звонку рендом бабки репортируют, а тут чуть МЕЖДУНАРОДНУЮ станцию не разбомбило, а все ебальники захлопнули и молчат.


Просто для понимания
https://www.hou.usra.edu/meetings/orbitaldebris2019/orbital2019paper/pdf/6001.pdf

>The ISS impact database is maintained at the NASA/JSC by the Hypervelocity Impact Technology (HVIT) group. The database contains over 1,400 records of impact damage from ground-based observations of space-exposed hardware returned from ISS


>MMOD damage areas of interest acquired from EVA and IVA sources mentioned here are collected in an image database managed by the Image Science and Analysis Group at the Johnson Space Center (JSC). There are currently 380 records tagged as “MMOD” in the database, although it should be noted that some of these records include multiple MMOD damages (i.e., there are more than 380 MMOD impacts represented in this database).


Т.е. в станцию постоянно прилетает какая-нибудь хуйня, повреждая оборудование, 1800 прилётов это лишь то что задокументировано. Солнечные панели, радиаторы, электрические силовые агрегаты, трубки охлаждения, даже иллюминаторы. Всё что угодно. За время существования модуля Купол ему в окна прилетело два болтика, из которого второй в 2014 был весьма крупным, но стекло выдержало. После чего защитные створки стали держать закрытыми, и открывать только по необходимости.

>>65184

>И сами же и выбрасывают во время ВКД.


В 2008 кажется потеряли сумку с инструментами, она улетела в антинормаль (в наихудшем направлении). Соответственно через полвитка прилетела обратно. Но всё обошлось.
300 665218
>>65199

>но по-моему даже эта пикча явно показывает что планету видно больше чем точка, не?


Я могу обыкновенным фотоаппаратом снять любую достаточно яркую звезду таким образом, что будут видны не только экзопланеты, но даже протуберанцы. Что это докажет? Только то что объектив у меня так себе.
Вот и картинка, которую ты притащил, ничего не доказывает сама по себе, чтобы делать какие-то выводы необходимо знать параметры телескопа, параметры съемки и прочее, а потом еще расчетами все обосновать. Так какого хрена ты делаешь выводы о каком-то там разрешении и не точечности источника?
768677676786876767.mp43,2 Мб, mp4,
1280x720, 0:15
301 665404
Где искать новую инфу по теме космоса астрономии?, уже все пересмотрел, елозят одни и те же темы по 10 раз, может есть иностранные популярные источники?
302 665411
>>65404

>по теме космоса астрономии


В твиттере, на дваче в новостетреде. А на деле тебе нужно очертить зону интересов потому что космос и астрономия изучают вообще всё - планеты, астеройды, газ, звёзды, пустоту, магнитные поля, вулканы, недра, галактики пиздец все мы находимся в космосе, земля наша в космосе и часть космоса, космос это всё, поэтому нужно очертить тему внутри космоса которая тебя особо интересует
303 665413
>>65404
Мне помогают каналы
Scott Наше Всё Manley
Fermilab
PBS SpaceTime
304 665446
>>65404
По планетам и результатам АМС, пролистываю Icarus по верхам, в интересное вникаю.
305 665484
а когда черные дуры сливаются, в энергию волн уходит их масса или потенциальная энергия падения? Или в данном случае это эквивалентно (подобно тому, как масса ядра атома больше массы компонентов)? Крч, Итоговая чд легче родительниц?
306 665486
>>65484
Массы.
307 665487
>>65404
Заходи на arxiv.org выбирай astrophysics и new. свежайшие статьи, каждый день десятки новых. много скучного мусора типа "чето там намерили, спектр линии CaII в галактике YOBA901488", но 1/10 статей ниче таких. статьи все на глицинском
308 665494
>>65484

>в энергию волн уходит их масса



А как связана масса и "энергия гравитационной волны"? Каким уравнением?
Если вопрос показался сложным, то другой: Почему ты решил, что гравитационные волны переносят энергию?
Стикер63 Кб, 450x450
309 665495
>>65494

>Почему ты решил, что гравитационные волны переносят энергию?


Ну охуеть, лазеры в интерферометре без затрат энергии меняют свой путь, волшебство что ли?
310 665500
>>65495

>Ну охуеть, лазеры в интерферометре без затрат энергии меняют свой путь, волшебство что ли?



О каких "затратах энергии" ты говоришь? Ты уверен, что понимаешь общепринятую сейчас концепцию того что такое гравитация и гравитационные волны?
311 665505
>>65500
Ты вокруг да около не ходи, а сразу поясни что к чему: "так и так залупу тебе за воротник искажение пространства происходит без затрат энергии потому что твоя мамаша очень жирная", а то макабу шатаем попусту.
15145843088380.jpg39 Кб, 720x533
312 665511
>>65494
Следи за руками. Человек задал тупой вопрос >>65484 : "куда уходит лишняя масса чёрных дыр при слиянии, и вообще уходит ли". Он ни-ху-я не утверждал. А ты вместо того чтобы ответить на тупой вопрос в треде тупых вопросов, начинаешь его пинать и самоутверждаться на нём, словно он что-то заявил. При этом не внеся никакой новой информации, а лишь задавая туманные вопросы. Теперь все думают - нихуя ты умный! Крутой финт, да?
313 665512
>>65505
Да это местный дурачок с реддит спейсингом, который начинает задавать тупые вопросы в ответ на тупые вопросы. Он тебе нихуя не пояснит, т.к. сам нихуя не знает.
314 665518
>>65494
>>65500
Ёбаное чмо, тебе пора ссать на твою умную рожу.
https://en.wikipedia.org/wiki/Gravitational_wave#Energy,_momentum,_and_angular_momentum

>Water waves, sound waves, and electromagnetic waves are able to carry energy, momentum, and angular momentum and by doing so they carry those away from the source. Gravitational waves perform the same function. Thus, for example, a binary system loses angular momentum as the two orbiting objects spiral towards each other—the angular momentum is radiated away by gravitational waves.

315 665519
Двач, а что за звезду/планету сейчас видно в Москве в полночь, если смотреть на восток? Небо чистое, но рассмотреть звёзд вокруг нее у меня не получается. Крупная и яркая в сравнении с другими на небе.
image.png1,1 Мб, 3369x1434
316 665520
>>65519
Скорее всего Юпитер.
https://stellarium-web.org
317 665522
>>65520
Спасибо, анон.
318 665526
>>65505

>Ты вокруг да около не ходи


Тебе прямым текстом же сказали, что гравитация это не сила и не поле и что "в энергию волн уходит масса" это абсурд. Пространство не плоское, оно "искривлено" и то, что мы называем гравитацией это движение по прямой, просто в искривлённом пространстве-времени. Это Общая Теория Относительности.
319 665527
>>65526

> "в энергию волн уходит масса" это абсурд


>Это Общая Теория Относительности


https://sci.esa.int/web/lisa/-/31417-gravitational-waves
Просвещайся.

> It was not until the late 1950s that some relativity theorists, H. Bondi in particular, rigorously proved that gravitational radiation was in fact a physically observable phenomenon, that gravitational waves carry energy, and that as a result a system that emits gravitational waves should lose energy.


Если бы гравиволны не могли переносить энергию - их нельзя бы было сдетектить и обнаружить в принципе. Весь движ Торна со товарищи по поиску волн начался именно с теоретического результата выше.
320 665529
>>65526
Бля, чё ты несёшь вообще, как раз ОТО и предсказывает гравитационные волны, и что они уносят энергию и момент импульса.
321 665530
>>65526

>Тебе тупым вопросом же ответили


Фиксанул.
322 665533
>>65527

>Если бы гравиволны не могли переносить энергию - их нельзя бы было сдетектить



Ок, тогда назови уравнение с помощью которого можно рассчитать "энергию гравиволн" и единицу измерения этой энергии.
323 665534
>>65529

>как раз ОТО и предсказывает гравитационные волны


Вот типичный итого того, что в школе не преподается ОТО.
324 665537
>>65529
Ты бы хоть википедию просчитал, раз с ОТО не знаком.

https://ru.wikipedia.org/wiki/Общая_теория_относительности

>Общая теория относительности (ОТО; нем. allgemeine Relativitätstheorie) — общепринятая в настоящее время теория тяготения, описывающая тяготение как проявление геометрии пространства-времени. Предложена Альбертом Эйнштейном в 1915—1916 годах.



>В этой теории постулируется, что гравитационные и инерциальные силы имеют одну и ту же природу. Отсюда следует, что гравитационные эффекты обусловлены не силовым взаимодействием тел и полей, находящихся в пространстве-времени, а деформацией самого пространства-времени, которая связана, в частности, с присутствием массы-энергии.

325 665539
>>65533
Опять туманные пассивно-агрессивные вопросы, вместо того чтобы просто признать что ты неправ? Поссал тебе на лицо, жоповилец.
https://arxiv.org/pdf/gr-qc/9712019.pdf
Глава 6, уравнение тензора энергии-импульса. Оправдывайся. Хотя лучше не надо, просто выйди из треда и не возвращайся в него никогда.
Также в этом учебнике
http://www1.jinr.ru/Archive/Pepan/1975-v6/v-6-1/v6p1pdf_obzory/v6p1_06.pdf
страница 251, твоё уравнение в более привычной форме.

>>65534
>>65537
Ты сам-то изучал ТО? Нихуя ты не знаешь, только квохчешь в треде тупых вопросов, как всё хуёво в школе (проиграл с ОТО в школе, кстати). Как и сказано в линке в посте выше, гравитационные волны меняют дистанцию между источником и наблюдателем, в локальной СО это выглядит как энергия, привносимая в систему извне.
Читай также вот это https://ru.wikipedia.org/wiki/Аргумент_липкой_бусинки
326 665540
>>65539

>Как и сказано в линке в посте выше


Вот в этом >>65527
327 665541
>>65539

>проиграл с ОТО в школе, кстати


Почему?
328 665542
>>65539

>Опять вопросы


Да, а ты надеялся, что тебе их не зададут?

>Глава 6, уравнение тензора энергии-импульса.


Это уравнение по твоему мнению "описывает энергию гравиволн" ?
Тогда ты легко назовешь единицу в которой измеряется эта энергия, я прав?
329 665544
>>65542
Это уже троллинг тупостью пошёл. Тот анон прав, тебе надо съебать и не возвращаться.
330 665545
>>65494
В вики написано, алсо тут где-то https://www.youtube.com/watch?v=--6WAmBHmiY не кто-нибудь, а САМ штерн сказал, что массу уносят, ну я хз
331 665546
>>65541
Дурачок же, дурачок видит что-то непонятное и немедленно проигрывает.
332 665547
>>65541
Да на кой хуй ОТО в общеобразовательной школе? Школьники и так перегружены предметами, у них не только физика, поинтересуйся сколько часов уделяется той же мировой истории и что потом получается. В специальной физмат - да, конечно.
333 665548
>>65544

>Это уже троллинг тупостью пошёл.


Этот умник >>65539 утверждает, что "уравнение тензора энергии-импульса" это уравнение с помощью которого можно рассчитать "энергию гравиволн". Раз он в этом уверен, то он легко может называть единицу измерения этой "энергии гравиволн". Но что-то мне подсказывает, что он сольется в визг.
334 665550
>>65547
Поразительно, не знал.
403K.gif3 Мб, 480x270
335 665552
>>65548
Я тебе ничего называть даже не собираюсь, просто обоссу ещё раз твою самоуверенную морду, прочитавшую всё что я дал для образования, но так и не способную признать неправоту, виляя анусом.
336 665553
>>65548
энергия в джоулях, ты бы чето смешное придумал
337 665554
>>65547

>Школьники и так перегружены предметами


Физикой 19 века, да. А потом удивляемся, почему люди не знают о современном представлении гравитации и считают, что гравитация это сила и гравитация:

>>65529

>уносят энергию



>>65545

>массу уносят

151G.gif775 Кб, 350x200
338 665555
>>65554
Держи ещё, мартышка, сама ротор от дивергенции отличить не способная.
339 665556
>>65552

>Я тебе ничего называть даже не собираюсь, просто обоссу ещё раз твою самоуверенную морду



Ты сейчас при всех поссал в лицо Эйнштейну и всем образованным людям знакомым с общепринятой ныне теорией гравитации - ОТО.
Тебе не стыдно?
340 665561
>>65554
Ты аргумент липкой бусинки комментировать собираешься, шизло?
341 665577
Существует ли Планк? Или это миф? ПЛАНК что вообще за фамилия такая лол, напоминает криво сляпанный конструкт по типу S.T.A.L.K.E.R.
придерживаюсь варианта#2
342 665586
>>65577
Макс Планк? Не существует, он умер лет 70 назад
image.png97 Кб, 400x400
343 665591
Я правильно понел что температура это колбашение атомов, когда они газни бульбни то они замедляются/меняют направление просто впидорашиваясь друг в дружкины магнитые поля, а в молекуле они уже внутри магнитной ловушке и шатаются внутри ее потанцевала и постоянно меняют направление и эта смена направления, т.е. ускорение туды-суды трати энерхию и эта энергия выплевывается тепловыми фотонами?
344 665593
Физический вопросдля простоты на наглядном примере: допустим с Земли стартует космический корабль на который установили очень яркий световой маяк, который загорается и гаснет с постоянной частотой,скорость распространения света постоянна, скорость корабля так же постоянна. Поначалу очевидно задержки между временем отправления и получением светового сигнала почти не будет, но через некоторое время она должна быть существенная. Вопрос - как будет это выглядить для наблюдателя с Земли? Частота мигания маяка будет постепенно замедляться? Или как это работает?
345 665594
>>65593

>скорость корабля так же постоянна


Частота не изменится никак от слова совсем.
346 665603
>>65554
>>65537
Ты бы хоть почитал, в каких условиях гравитационные волны возникают что бы не позорится. Хинт - вот когда просто есть тело и пространство искривлено, никаких волн не возникает.
347 665609
>>65591
Температура это общая термодинамическая мера разных физических явлений. Она может быть пропорциональна энергии частиц в плотном веществе, скорости отдельной частицы, ебошащей через космос, у плазмы бывает две разных температуры - электронов и ионов. А бывает ещё температура излучения - точнее не самого излучения, а оно соответствует телу с такой-то температурой. Например реликтовое излучение выглядит как тело со средней температурой 2.7К, хотя тела там нет.
348 665617
>>65609
Схуяли это там нет тела? Это светит рекомбинирующий водород и гелий с границ видимой Вселенной, просто пока фотоны летели, их растянуло с температуры ~3000K до 2,7К. Но это самое настоящее тепловое излучение, ничем не хуже излучения сковородки на плите.
349 665651
>>65603

>что бы


>позорится


Оно ещё что-то говорит о позоре
350 665726
>>65651
Срезал!
Pressurefedrocketcycle.png62 Кб, 800x982
351 665999
Вот есть значит реактивный двигатель, у которого есть тяга и удельный импульс. Почти всегда указывают значение для вакуума и уровня моря. Какие там графики? Можно тупо линейно интерполировать, или кривая загибается куда-то? А если допустим взлетаем с Венеры, как экстраполировать с 1 атм до 90? Есть ли предел давления, после которого тяга будет 0, либо она будет асимптотически к нему стремиться?
352 666001
>>65999
Зависит от формы сопла, у предназначенных для работы на уровне моря УИ в вакууме примерно на 10-15% выше чем у моря. Вакуумные очень хуево работают на уровне моря, на 50-70% хуже, скорее всего вообще взорвутся. На Венере примерно то же самое будет, двигатель просто не заработает.
353 666002
>>66001
Раптор вакуум не взорвался, интересно сколько у него было импульса и тяги
354 666033
>>65999
Двигатель работает на разнице между давлением в КС и наружным. Если у тебя на Венере будет меньше наружного, то он в себя будет всасывать нахуй. Но КС, рассчитанная на разницу давлений в 90 атм на Земле, на Венере выдержала бы 180 - если бы не температура.

На венере при нескольких сотнях градусов никакое топливо не будет храниться нормально, может только та высокотемпературная хуйня для SR-71, хз только что окислителем ставить и как сопло охлаждать
355 666038
>>66002
У него вроде сопло было по форме как у обычного
356 666079
>>66033
ТТРД же сможет работать?
357 666082
>>66079
А какая разница, он тоже на разнице давлений пашет. Если его не сожмёт в какаху и горючка при 480С + доп.температура не разложится (а ей пизда придёт) - то в теории можно было бы юзать.
358 666083
>>66082
Ну вот как раз я ТТРД и вижу чем-то что как-то смогло бы. Наполняешь бочку крепкой такой смесью которая только в автоклаве работает и засылаешь.
Чето захотелось на Еву сгонять в огурцах.
359 666110
>>66083
Инфа 100% что нет такой смеси, а если и есть, то её распидорит при включении, несмотря на абляционный слой.
Для взлёта с венероподобной планеты лучше юзать нетрадиционный старт, например вертолётный или дирижаблевый. Это же по сути "океан", там всплываешь скорее. Причём неясно, как сохранить горючку целой на поверхности, разве только стыковаться с летающей платформой, но на таких высотах на Венере ветер ебанутый вокруг шарика.

Реалистически говоря, это всё манядомыслы уровня колонизации поясов ядерными звездолётами.
360 666114
Я не знаю где такое спрашивать но спрошу на своём любимом спесаче(если он не трансмутировал за время моего отсутствия.)

Есть ли где карты непрерывности лесных участков? То есть карта где показаны участки леса без дорог, разного рода просек, рек и т.д. ?
361 666115
>>66114
Не знаю насчет буквально непрерывности (это какую-то производную надо считать), но есть https://www.globalforestwatch.org/ , который показывает изменение лесного покрова Земли и текущее состояние. Две галки сбоку снять и получишь мгновенный снапшот текущего состояния.
А ещё вроде в OSM можно как-то отфильтровать по лесам, и в GIS разной степени платности.
362 666117
Существуют ли прикидки хотя бы уровня «на пальцах», сколько поколений людей должно родиться, вырасти, родить и вырастить в условиях микрогравитации, чтобы приспособить органы к ней?
363 666118
>>66117
Нет. Вообще неизвестно, будет ли жизнеспособен ребёнок, выношенный и рождённый в условиях микрогравитации.
364 666121
У jwt цветные фотки будут?
365 666123

>jwst

366 666124
>>66121
Будут.
367 666138
>>66110
Хуй с ней с Венерой. Есть Титан. Что будет с импульсом и тягой на поверхности? Может встречал кто-то промежуточные данные, типа импульс/ тяга в вакууме, 1 атм и 0.5 атм? Нужны опорные точки для интерполяции
368 666191
>>65999
>>66138
https://ru.wikipedia.org/w/index.php?search=Удельный импульс&title=Служебная:Поиск&ns0=1
тут даже формула есть с внешним и внутренним давлением, держи
369 666393
1) Что за прыщ на теле Ганимеда на фото? Это что, гигантская гора? Какого она размера навскидку? Больше марсианского Олимпа?
2) Откуда инфа, что на Титане пески из платика? Что под этим подразумевается?
3) Предположим, люди успешно запульнули колонистов на планету земного типа в обитаемой зоне, обладающей ещё и полностью идентичную по химическому составу, по температуре и по плотности атмосферой, вот только там почти вся поверхность - вода. Как в таком случае будет спасаться корабль с колонистами? Как они всплывут со дна и доберутся до берега?
3.а) Размышляя о вопросе (3), в голове мелькнула идея основывания подводной колонии в условиях довольно ограниченного запаса материалов. Допустим, есть там запчасти и допматералы (похуй, как), но находясь уже под водой, как обеспечить воздуходышащим людям воздух, если ты глубоко? И беда в том, что даже при постоянных открытиях-закрытиях шлюзов воздух всё равно будет постепенно утекать. Но что если тогда вообще убрать шлюзы, гермодвери, затворы, люки т.д. совсем, оставив эм входную трубу полностью открытой?Поэтому смотри пикрил 2, на котором 3 варианта при которых возможно такое, что внешнее давление среды - то есть толщи воды - одновременно зажмёт воздух, не давая ему уйти, и оставит выход. Естественно, вода будет давить и пытаться занять объём менее плотной среды дыхательного воздуха, но насколько? Можно ли как-то просчитать соотношение глубины и давления воды извне и длину части трубы, отданной под затопление, чтобы получить равновесие между водой и воздухом, чтобы в получившемся воздушном кармане можно было уютно сычевать большой колонией или в одиночку?
370 666398
>>66393
Охуеть, дегроид с двача изобрёл водолазный колокол

>Можно ли как-то просчитать


Нет, нельзя - это высшая математика

>Как в таком случае будет спасаться корабль


От кого спасаться-то, бля?
371 666399
>>66393

>1) Что за прыщ на теле Ганимеда на фото? Это что, гигантская гора? Какого она размера навскидку? Больше марсианского Олимпа?


Это Область Галилео
https://en.wikipedia.org/wiki/Galileo_Regio
372 666436
>>66117
Никто никогда не беременел и тем более не рожал в в микрогравитации.
373 666473
>>66436
мыши рожали, нет?
374 666530
>>66399
Это кратер, просто свет так падает.
375 666586
>>66473

> мыши рожали, нет?


Ну рожали и рожали. Чего бухтеть-то?
Снимок.JPG31 Кб, 558x415
376 666637
Как правильно переводится ПОККОКМОК или ПОЦКОЦМОЦ?
377 666639
Мне в школе говорили что космонавты там еба перегрузки испытывают, и здохнуть при взелете это на раз два? Тогда как сайчас все эти безонсы-хуезонсы пенсовые летают?
378 666642
>>66639
Ебовые перегрузки испытывают при баллистическом входе в атмосферу с орбиты, когда падаешь как камень - вот на центрифуге в основном для этого крутят. Большинство кораблей сегодня в штатном режиме входит в атмосферу в режиме планера, там перегрузки намного меньше, и в противоперегрузочном кресле их держать проще чем лётчикам. Вон Гленн на Шаттле в 77 лет летал. А Нью Шепард суборбитальный и высота там всего около 100 км, там траектория баллистическая, но энергии мало, поэтому не особо плющит и на малое время. В том же Меркури 7 в 61 году Шепарда вжало на 11G, но совсем на чуть-чуть. И то высота у него была больше в пару раз, и капсула Меркури была мелкой и тормозила плохо, не то что Нью Шепардовская с широкой жопой.

Ну и плюс лысый видимо не такой уж и хлипкий как выглядит, думаю. А уж тем более баба, даже в 83, её ж в реальные астроняки выбирали и тренили. Чак Йегер на истребителе ебошил в 90+ лет. правда он вообще шестиметровый сверхчеловек был с железобетонными яйцами, уровня гагариных с армстронгами

Мне больше интересно что бы было если сработала САС во время полёта. Пенсов бы наверняка плющнуло нехило - там
379 666643
>>66642

>Пенсов бы наверняка плющнуло нехило - там краткие перегрузки могут быть на десятки g, даже опытных космолюдей может травмировать неслабо


фикс
3a643b45080849218205f2a430226666.jpg484 Кб, 1680x1050
380 666652
Есть ли смысл делать вытеснительную подачу >>65999 водяным паром?
Возить его легко, кипятить в теплообменнике тоже легко. А давления там пиздец. http://www.fptl.ru/spravo4nik/davlenie-vodyanogo-para.html Перегретый до 370С 200 атмосфер выжимает.
381 666669
>>66530

>Это кратер, просто свет так падает.


Что за бред ты несёшь? Свет падает слева напрвао, поэтому справа теневая полусфера. Ну а раз так, и если бы это был кратер, то тогда тень бы была слева,а освещённая часть справа. Но тут на снимке тень справа, как и на самой планете. Значит, там не вогнутость, а выпуклость, то есть не кратер, а гора.
16152987033400.jpg346 Кб, 1421x1066
382 666670
383 666688
Что такое реликтовое излучение?
Я правильно понимаю что это якобы излучение от того что было 13 млрд лет назад, что-то похожее на галактики которые находятся в 12 млрд с/л от нас
384 666689
>>66688
Это излучение от содержимого самой вселенной, когда она расширилась достаточно, чтобы раскукожиться из сплошного батиного супа в более-менее прозрачное пространство. Звезд тогда ещё не было, они позже свернулись в шарики из этого супа.
385 666694
>>66688
Когда Вселенной было 380 тыс. лет, вещество перешло из состояния плазмы в состояние газа, протоны, нейтроны и электроны объединились в атомы, Вселенная стала прозрачной для излучения, которое отцепилось от вещества и смогло свободно путешествовать. Никаких галактик и звезд тогда конечно же не существовало.

Момент объединения протонов, нейтронов, электронов в атомы и освобождения излучения называется рекомбинацией. Это самое излучение и есть реликтовое излучение. Оно дает нам по сути детское фото Вселенной, ведь на момент рекомбинации, как уже говорилось, ее возраст составлял всего 380 тыс. лет.
386 666696
>>66688
и кстати из за расширения вселенной это уже не в 13 млрд световых лет от нас, а в ~45 млрд, т.к. вселенная вполне умеет расширяться быстрее скорости света на больших расстояниях. из чего следует, что большая часть видимой вселенной уже абсолютно недостижима для нас, даже если научимся разгоняться до 90% скорости света
387 666697
>>66652
у тебя бак 200 атмосфер не выдержит и буквально жидко пукнув порвется. к тому же топливо в баке закипит от такого пара
388 666702
>>66652

>Возить его легко


Это по сравнению гелием-то?

> кипятить в теплообменнике тоже легко


Ты будешь его кипятить двигателем? А запускать как?

И к тому же гелий инертен, а вода нет.
389 666703
>>66702
а почему гелий, а не водород? при том же давлении масса меньше
390 666707
>>66703
Гелий инертен, а водород нет.
391 666725
>>66707
ладно, а почему не самоподдув?
392 666726
>>66703
Яб посмотрел на наддув окислительного бака водородом.
393 666828
>>66726
слабоумие и отвага! он же холодный
394 666859
>>66726
Сами топите урановые ломы в ртути.
395 666905
>>66726
Ну на некой ракете бак с амилом наддували гептилом, а бак с гептилом - амилом. Но чуть-чуть, так что не ебанёт.
396 667018
>>66905
А ето как? Они же гипер-голые, ебошат при контакте
397 667065
>>67018
Вот и выделяется горячий азот для наддува
398 667606
https://youtu.be/9pbra_FYn7k

Послушал релейтед и ОХУЕЛ.

TL;DR:
НАСА собрались сфотать детали поверхности экзопланеты, удалённой на 100 световых лет, в разрешении 1000x1000 пикселей, использовав для этого ни много ни мало гравитационную линзу солнца, фокус которой находится на расстоянии 600 а.е. (в 4 раза дальше Вояджера), отправив туда телескоп весом 70 кг с помощью солнечного паруса. Предполагаемая скорость разогнанного аппарата должна составить 25 а.е. в год, время полета составит ~25 лет.

Это самая шизоидная хуйня, что я слышал со времён высеров Маска про город на Марсе.
Неужели есть шансы на успешную реализацию?
399 667613
>>67606
Почему нет? Гравилинзирование, интерферометрия и солнечные парусы это вполне работающие технологии, просто в сочетании их никто пока не применял.

Так-то замечательных идей овердохуя, если их всех реализовывать - получилось бы несколько десятков бюджетов JPL. Например растянуть несколькими роверами параболическую сетку над лунным кратером на обратной стороне, и получить огромный зенитный радиотелескоп в естественной зоне радиомолчания, сравнительно малыми усилиями. Или робокапсулы с радиоизотопным нагревом, чтобы проплавить льды Энцелада и добраться до рачков внизу.

Проблема только в том, что это директорат NIAC, т.е. не утверждённый проект, а просто хотелки, которые выдвигают на рассмотрение, и хз там сколько ещё тонкостей и НИР до постановки в разработку пройдёт. Так что шансы реализации равновероятно делятся между десятками других заебательских идей.
400 667615
>>67606
Жаль только этот телескоп на ограниченное число планет можно навести будет.
401 667616
>>67615
На одну звёздную систему. Я крайне удивлен, что это вообще физически возможно - сфотать экзопланету в разрешении 1 мегапиксель. На земле не всякий телескоп в таком разрешении Марс сфотает.
402 667629
>>67606

>НАСА собрались


лол это же просто концепт, примерно как космический лифт
для линзирования нужно расстояние в 550 раз больше чем от солнца до земли, а запущенные в 80х вояджера улетели на 150 ае, новые горизонты 50 ае
403 667647
>>67629
И хули? Разгон на солнечном парусе, с маневром максимального сближения с Солнцем.
Потом 30 лет полёта.
Вполне адекватные сроки.
Плюс сама машина не должна быть саерхдорогой, плюс такие телескопы придётся горстями запускать.
Другое дело, что проблема не техническая и даже не финансовая, требуется железобетонно интересная, читай живая, планета для наблюдения.
Так что пока всякие уэббы и очень большие йобы не дадут точного биомаркера у экзопланеты, никто никуда не летит.
404 667654
>>67647
А останавливаться как? Или фотать только в момент фокусировки - условно месяц, а потом и хуй с ним, пусть это говно улетает?
Мало того, что он лететь будет десятилетия, так ведь этот месяц может искомая планета оказаться позади звезды. Или какое-то пылевое облако вылезти.
А, если, вдруг, на нужной планете, есть атмосфера, то может быть облачно в этот месяц - и опять ничего не увидишь.

Довольно рискованная космическая программа просто по наблюдениям, не говоря уже о неведомом телескопе, способ навести его в миллионные доли градуса, способ передать с такого расстояния на Землю и прочее
405 667655
>>67654
Никак. Пролетел и все.
406 667664
>>67654

>А останавливаться как? Или фотать только в момент фокусировки - условно месяц, а потом и хуй с ним, пусть это говно улетает?


Так и есть.
407 667668
>>67647
столько новых компонентов в одну миссию никогда не сделают
а на тестирование еще лет 10 или больше
408 667674
>>67668
Ну так-то да, надо ещё планету найти нормальную, для этого нужен уеб и другие специализированные телескопы.
Причем планета прям пиздатая нужна, земного типа, с атмосферой, в зоне существования жидкой воды и желательно ещё со странными пуками в спектре.

Пока таких нет.

Но мне больше интересно, когда уже какой-нибудь кубсат мощно разгонят парусом, чтобы он до Плутона за год метнулся.
409 667676
>>67629

>примерно как космический лифт


Разница всего лишь в том что лифт нереализуем и туп по определению, а эта хуйня реализуема и не тупа. И нет, это не концепт, а предложение в NIAC, там нужно более серьёзное обоснование чем "ну это, я тут нанотрубки посчитал на салфетке", там нужен PI, коллектив, обоснование, примерный план, в общем миссия должна быть реализуемой.

>>67654
А как останавливались New Horizons? Да никак, это пролётная миссия. Можно такой зонд нагрузить ещё чем-нибудь, чтобы изучать окрестности Солнечной системы, если массовый бюджет позволит.
410 667677
А как вообще работает гравитационная линза, если весь свет загораживается самим массивным объектом?

Или она улавливает тот свет, что проходит рядом с массивным телом?
Его достаточно для наблюдения?
411 667678
>>67677
Тот свет что рядом. Достаточно, но нужно только закрыть само тело хуйцом, иначе оно будет ярче чем объект. Ещё он придёт люто искажённый, придётся по лесу формул и при помощи лютого кластера восстанавливать истинную картинку.
412 667771
>>67606

>НАСА собрались


Успокойся, никто ничего не собирается. Разговоры о возможности уже лет пятьдесят идут, а как сделать - никто не знает
413 667772
>>67606
проблема в том, что у солнца есть корона. отдалиться нужно минимум на 2000 а.е. там не идиоты, они это учтут
414 668039
Какие новые возможности откроет ТЭМ? Пускать всякие вояджеры и новые горизонты на край системы вроде уже и сейчас получается.

Дешевле пускать грузы к Марсе и Луне? Хуета какая-то, неинтересно.
415 668047
>>68039
Новые возможности для попилов, пожалуй. Хотя восточный наврятли что-нибудь превзойдет.
416 668060
>>67606
Пока не собрались, достойных целей пока нет. Поскольку телескоп строго одноразовый, зато дает такую крутую картинку его стоит рассматривать как реалистичный аналог пролетной миссии.
Тут нужно понимать одно, не будет достойной цели - не будет и миссии. А будет достойная цель, деньги выделят. А критерий достойности банален, биомаркеры. На планету с биомаркерами миллиардов не пожалеют.
По реализации в целом все понятно и упирается в бабло, вариантов отправки с требуемой скоростью что бы добралось за осмысленное время куча, там и ЭРД с реактором, и маневр Оберта у Солнца, и паруса все это уже посчитано.
417 668074
>>68039
ТЭМ это мощный источник электроэнергии в космосе, которой никогда много не бывает. Радары жрут овердохуя. ЭРД жрут овердохуя. Системы связи жрут овердохуя. Практически любую деятельность можно подвязать на электропотребление - от связи до мемного производства кристаллов на орбите - главное чтобы она была в наличии. В общем эта теха фундаментально полезна.

>Пускать всякие вояджеры и новые горизонты на край системы вроде уже и сейчас получается.


А например пускать орбитеры к лунам Сатурна/Юпитера нет, гугли JIMO.
418 668099
>>68060
Какие могут быть биомаркеры? Следы органики в спектре атмосферы?

Какие телескопы запущены для поиска биомаркеров?
419 668115
>>68074

>ТЭМ это мощный источник электроэнергии в космосе


А Конаныхин - мощный источник электроэнергии пидорашек. Вот только его мантры не сбываются.
420 668137
>>68099

>Какие могут быть биомаркеры? Следы органики в спектре атмосферы?


Ну самое банальное это кислород в больших количествах. Но вообще любые вещества которые нестабильны на длительных периодах времени и путей синтеза которых на планетах земного типа быть не должно, причем желательно несколько разных.

>Какие телескопы запущены для поиска биомаркеров?


В данный момент никаких, но у JWST одна из задач это снятие спектров атмосфер экзопланет.
421 668138
>>68099
>>68137
Ну и да, немного спектры уже снимают и сейчас, но это единичные случаи на всяких горячих юпитерах.
422 668166
>>68115
Кто это? Не в курсе ваших русскоязычных блогеров, или на кого вы там дрочите у себя
423 668170
>>62981 (OP)
Какой телескоп стоит купить ньюфагу, если есть только хрущевка в дс2? Хочу видеть кольца Сатурна с просветом меж ними! Сколько будет стоить самая дешёвая версия для этой цели?
424 668174
425 668195
>>67606
Нахуя городить такую ебанину, если ещё терраскоп не построен?

Слои атмосферы земли выше тропосферы - практически идеальная линза. Землю с тропосферой закрываем непрозрачным экраном, ловим лучи с окружности, получаем апертуру в 13000км и светособирание как у 1400км зеркала, что в любом случае на много порядков превышает все самые распиздатые телескопы, и на съёмку экзопланет этого хватит с запасом. Также получаем обзор вдоль орбиты, а если у нас в запасе дохуя ксенона на смену наклонения - обзор на все 360 градусов во все стороны. Орбита нужна в ~4 раза дальше Луны, т.е. менять наклонение там довольно дёшево.
426 668224
>>68195
Хуй знает, возможно, атмосфера будет давать дохуя искажений.
427 668233
Назовите мне ВСЕ известные вам фильмы на комическую тематику.
Хочу из всех предложенных названий, подобно отделению зёрен от плевел, отделить то,что смотрел, от того, чего не видел, и то, чего не видел, таки посмотреть.
428 668287
>>68233
Сквозь горизонт
429 668288
Если на момент смерти вселенной вернуться в прошлое, я останусь в живых
430 668295
>>68288
Ну, круто тебе.
431 668297
>>68224
А возможно не дохуя, или их можно будет компенсировать. Нужны исследования этого вопроса.
432 668298
>>68233
Геи-нигеры из открытого космоса
433 668358
>>68233
луна 2112
одиссея кубика
интерстелар
434 668368
чем определяется время на планете?
если мы возмём далёкий чуждый мир, как нам там считать часы, дни, недели, месяцы, годы?
напрмер, мир вокруг б0льшего солнца, чья орбита как орбита обитаемости как орбита юпитера в солнечной системе. может ли быть у такой планеты сутки быть восемнадцати или семидесятичасовыми? а в неделе 5 дней или 13? а в месяце 20 дней? а в году 100 дней или 666 дней? что и как это всё будет определять? математика?
435 668382
>>68368
Цикличностью чего либо. Погодные сезоны - год. Отливы-приливы, светило в небе - месяц. Птички поют-спят, цветы распускаются - сутки. Биоритм у человека, насколько я помню, заложен генетически. Придется разбивать время на земные часы и-или сутки. Или геном менять.
436 668391
Почему вместо всех этих наук ядерных буксиров, восточных и ангары с зенитом и федерациями нельзя было оптимизировать производство союзов чтобы их можно было делать по 50 штук в год а потом задемпинговать нахуй весь международный рынок, засрать нахуй всю орбиту и потом забить на этот ваш космос? Просто сказать что не нужон, и все эти гонки прекратились бы сразу? Неужели всегда нужно делать все максимально по уебански чтоб был повод просить бабло на переделывание и допиливание хуйни без задач?
437 668392
>>68368
если типа альт мирок то календарь будет по солнечным/лунным циклам впоследствии уточненным до значений звёздного года, недели вообще не факт что будут, часы будут делённые на двое сутки плюс ещё множитель на основе главенствующей системы счисления, точно так же с минутами, секунды уже более современный конструкт, очевидно что будет соизмерим со скоростью произношения циферок
если же типо колонизация то год будет точно по периоду вращения вокруг солнца, делённому на количество суток в году, или нацело с добавлением убавлением дней или не нацело с добавочным временем в одном из дней если разница будет небольшой
часы минуты секунды будут метрическими
438 668393
>>68391
Потому что Союз узкоспециализированная маршрутка. Ты ничего не можешь засрать если не создаёшь рынок.
То бишь, чтоб стоить 50 Союзов в год, ты сначала должен продать места в космическом отеле на эти 50 Союзов.
439 668414
>>68391

>Почему вместо всех этих наук ядерных буксиров, восточных и ангары с зенитом и федерациями нельзя было оптимизировать производство союзов чтобы их можно было делать по 50 штук в год


1. Он и так за 50 лет заоптимизирован настолько, насколько это вообще возможно.
2. Там столько древнего легаси, что проще с нуля РН+КА сделать. Муск же сделал.
3. В ангарах с федерациями нет ничего сложного, если целью является игра в долгую с захватом мировой космической отрасли. В РФии исторически играют в быструю: спиздить, вывести в оффшор, сменить кресло, кинуть через хуй стрелочников.
440 668448
Не знаю куда писать, поэтому сюда. Был на МАКСе, утащил кучу бумаги, в т.ч. и по вашей тематике, список тут - https://drive.google.com/folderview?id=1ZFe0ExtcfJksAeFMCwOlrEdWfC__Juog . Что будет интересно - отфотаю подробнее, пишите.
441 668570
Спутники на низкой орбите тормозятся атмосферой и, в конце концов, падают. А мне интересно как это сопротивление было бы по ощущениям? С чем можно сравнить - типа как если высунуть ладонь из окна машины 120 км/ч или как поток из вентилятора? Высоту имею в виду больше 100 км.
Короче на какой высоте встречный поток стал бы заметен (например листок бумаги начал прогибаться)?
И второй вопросик - при входе в атмосферу, когда становится виден раскалённый газ, он какого цвета? (когда только-только свечение началось).
442 668573
>>68570

>А мне интересно как это сопротивление было бы по ощущениям?


Никак, вообще не почувствуешь. Только рука распухнет от низкого давления.

>Короче на какой высоте встречный поток стал бы заметен (например листок бумаги начал прогибаться)?


Если на первой космической - километров 100 где-то наверное. На 70 км он уже сгорит, а на 150 нихуя не будет.

>И второй вопросик - при входе в атмосферу, когда становится виден раскалённый газ, он какого цвета? (когда только-только свечение началось).


Раскалённое что угодно одинакового цвета в зависимости от температуры. Когда только-только начинает светится - тёмно-красного.
003.jpg121 Кб, 600x338
443 668574
>>68448
Это всё больше в /wm/. Точные ТТХ всяких фазированных решёток и головок самонаведения.
Но если есть инфа по "Зевсу" с ядерным буксиром - сканируй сюда.

>>68570

>А мне интересно как это сопротивление было бы по ощущениям? С чем можно сравнить - типа как если высунуть ладонь из окна машины 120 км/ч или как поток из вентилятора? Высоту имею в виду больше 100 км.


На 100км ты в плане перегрузки ничего не почувствуешь, но будет припекать от бомбардировки молекулами на 25М. Ниже ощущения будут, как будто под тобой море из сплошного упругого желатина, а тебя в этот желатин сверху вжимает великан. И чем ниже спускаешься - тем сильнее вжимает.

>Короче на какой высоте встречный поток стал бы заметен (например листок бумаги начал прогибаться)?


80-50км.
444 668579
>>68573

>Раскалённое что угодно одинакового цвета в зависимости от температуры


Значит в кино такой голубенький как газ в плите, это просто для красоты? А на самом деле сначала красный, потом оранжевый, жёлтый и, наконец, белый?
>>68574

>80-50км


Это ты так примерно почувствовал, или серьёзно?
Иначе я не понимаю как НАСТОЛЬКО слабый, неощутимый ветерок может подвинуть многотонный аппарат, что он за сутки сойдёт с орбиты и сгорит. (я понимаю, что без трения даже слабая сила создаёт ускорение, но всё же)
445 668586
>>68579

>в кино такой голубенький как газ в плите


Это в каком?

Не, ну вообще во время горения бывает и свет других цветов от химических реакций, собственно газ в плите голубой именно поэтому, а не из-за теплового излучения. Но при входе в атмосферу такого не будет.
446 668594
>>68586

>Это в каком?


Навскидку только Гравитацию припоминаю, но видел ещё где-то.
Ладно, при низком давлении вода кипит-замерзает, но ведь не мгновенно это происходит. И вот мысль - допустим мы на Марсе, и у нас бесконечное количество воды. Получится ли вырастить картошку на открытом грунте, если без остановки поливать грядки, холодную воду откачивать, а сверху ещё поставить мощные лампы, чтобы грели и не давали нарастать льду?
447 668599
>>68594
растениям кислород тоже нужон
448 668639
>>68233
фильмы аля фильмы катастрофы по росту градуса кринжа:
Аполлон-13
Время первых, Салют-7
Пассажиры
449 668641
>>68574
>>68579
не 80-50 км лол, там воздуха уже дофига. 50 км это практически поверхность Марса вообще, а на Марсе тормозят в еще нескольких десятках километров от поверхности, не вводи человека в заблуждение. для аэродинамического захвата ~90 км пригодны, линия Кармана, которая якобы от идеализированных самолетов считалась 100 км, шаттлы разблокировали аэродинамические поверхности на ~120 км. так что лист бумаги задолго до 80 км начнет трепыхаться
450 668642
>>68594
нет, на открытом грунте не выйдет. отсутствие кислорода, азота, перхлораты в почве, выкипание воды (в том числе внутри растений)
451 668723
>>632252 (OP)

Какой телескоп стоит купить ньюфагу, если есть только хрущевка в дс2? Хочу видеть кольца Сатурна с просветом меж ними! Сколько будет стоить самая дешёвая версия для этой цели?
DSC5693.JPG4,4 Мб, 5504x3096
452 668751
>>68574
Зевса нет, сори, но вот что есть, интересует?
DSC5698.JPG4,1 Мб, 5504x3096
453 668756
И еще одну нашел
454 668760
>>68723
в 130 мм апертуру с 900 мм фокусного расстояния на 4 мм окуляре (8 мм + линза барлоу) в принципе должно быть видно кольца с щелью и ТВЕРДО И ЧЕТКО Титан. сейчас противостояние говно, следующее в середине августа 2022 и чуть получше, так что у тебя почти год чтобы выбрать и купить
455 668816
>>68641

>линия Кармана, которая якобы от идеализированных самолетов считалась 100 км


Сам Карман в своей работе насчитал 80 с хером км, это потом переопределили и округлили и вообще извернули. Там нет какой-то резкой границы, это довольно нихуёвый континуум.
456 668817
>>68751
Смотр-В и Кондор-Е - не думаю что там что-то принципиально новое показано, чего раньше про них не слышали.

Вот про наноспутник с гелиороторным парусом - это интересно, да и про МРБ неплохо бы почитать подетальней, если там больше чем просто концепция, про которую и так известно.

>>68756
Заебца, вот это схороню, понятны возможности по определению орбит
457 668818
>>68817

>Вот про наноспутник с гелиороторным парусом - это интересно


А, понял, это про Лебедь всё. Интересно есть ли там какая-то новая инфа.
458 668854
>>68817
Оборотные стороны всего этого добра, но вроде инфы на них немного, сорри
459 668855
>>68854
Блин, еще 2 забыл
460 668864
>>68855
Холодногазовый РБ, ебанись наркомания конечно.
461 668968
Есть ли spc-бугурт треды?
462 668969
463 668971
464 669002
Если коричневые карлики конвективны и при этом с вырожденым ядром, то что получается, у них материя туда-сюда гоняет и вырождается-развырождается постоянно?
465 669007
>>69002
почему развырождается? просто рано или поздно упоротый гибрид реакций синтеза-деления который их поддерживает выдохнется, т.к. кончатся радиоактивные вещества (подпинывающие это все гаммой, альфа-частицами и нейтронами) и легкие элементы способные на всякие интересные реакции
466 669047
Голубые карлики есть?
467 669048
>>69047
Есть, в Кремле один такой сидит.
468 669049
>>69007
ЩИТОО БЛЯТЬ? Чел это ты сделал тред "красного неба" и срал там абсолютно дегенеративной даунской хуйнёй? Похоже что ты. Так вот, если НЕ знаешь, то НЕ пиши. Весь твой пост это полная хуйня, ты абсолютный профан в коричневых карликах. Просто за-мол-чи.
469 669060
>>69049
во-первых успокойся и не бомби, к сети-троллю я отношения не имею, чини детектор. во-вторых что я не так сказал? рано или поздно коричневый карлик превращается в обычную тяжелую планету, но не в белый же карлик блжад
470 669294
Возможно ли общее воздушное пространство у двойной планеты, да ещё и такое, чтобы там, в этом общем воздушном пространстве, можно бы было дышать?
471 669309
>>69060
1 ты нихуя не понял о чём я спрашивал
2 ты вообще не шаришь за процессы в коричневых карликах
3 а теперь будь добр, замолчи
472 669315
>>69309
вместо того чтобы объяснить что не так с процессами внутри коричневых карликов ты просто требуешь замолчать? не замолчу пока не объяснишь. в чем я не прав? p-p цикл там и близко невозможен
473 669316
>>69294
Еслм брать очень близкую двойную планету, то да. Атмосфера будет иметь форму сплющенного шара. Но дышать между планетами ты не сможешь, даже если они очень плотные и прочные металлические, там все равно несколько тысяч км нужно радиус орбиты. Там и атмосферы считай нет.
964.jpg118 Кб, 963x700
474 669318
>>69294
Теоретически да, гугли полость Роша и обмен материе (между звездами правда)

>>69316
Там не будет эллипсоида, будет гантеля
475 669319
>>69315
Не люблю чсвшников, с уверенным видом "поясняющих" на деле полную хуйню. Я не буду ничего тебе объяснять: если никто кроме меня тебе не возразил, значит тут полные тупни, и тратить своё время на это я не буду. Пудри мозги глупеньким анонам дальше.
476 669320
>>69319
а может это ты тупень тогда с какой-нибудь альтернативщиной?
477 669330
>>69320

>высрал хуйню


>чел, ты...


>эээ мээ а может это ТЫ чел, ты??!


ха, лол, пока-пока
478 669344
>>69330

> высрался против упрощенного объяснения, которое было выдано из за игнорирования сути вопроса (затупил)


> спрашивают что не так?


> нискажу но ти дурак!1!1!

479 669654
Я правильно понимаю, что для человека кроме варп двигателя и чревоточин даже теоретически больше не существует способов перемещаться на космические расстояния быстро? То есть какой бы двигатель на каком бы топливе не придумали, этого будет все равно недостаточно чтобы не размазать кожаный мешок по креслу и при этом доставить его на Тау Кита быстро?
480 669656
>>69654

>кроме варп двигателя и чревоточин


Это тоже маняфантазии ебаные. Кротовых нор во Вселенной нет (по крайней мере в видимой части), а даже если бы и были, то для путешествий их невозможно было бы использовать. Во-первых, для того чтобы через нее прошел макрообъект типа звездолета, нужно экзотическое вещество с отрицательной плотностью энергии, чтоб кидать его пакетами в кротовую нору для удержания ее от схлопывания. Такого вещества не существует в природе. А во-вторых, даже если удалось каким-то чудесным образом раздобыть такое вещество, то это все равно никак не поможет пройти через кротовую нору. Там в горловине сингулярность, в которую звездолет неминуемо въебется и на этом все путешествие закончится. А варп-двигатель - это просто грезы. Никто его никогда не построит, потому что это скорее всего невозможно. Поэтому путешествие людей к звездам так и останется уделом научной фантастики.
481 669749
>>69656
Нахуя ты это высрал, если тебя о другом спрашивали?
482 669822
>>69654
Есть серьёзные основания предполагать, что пространство-время это не тянущаяся резинка, а сорт оф сверхтекучая жидкость. Явное свидетельство тому - запруфанный "водоворот" пространства вокруг нейтронных звёзд (см. frame-dragging, эффект лензе-тирринга), что невозможно с метрикой резинки, но возможно с метрикой флюида. Эффект предсказан в ОТО ещё Эйнштейном, но из-за доведённой до абсурда эфирофобии как-то подзабыт.

Более осторожный вывод: сверхсветовые джеты таки действительно сверхсветовые относительно удалённого наблюдателя, однако досветовые относительно местной метрики. Изучены джеты плохо, точного механизма полярного струйного течения нет, версия Мартина Риса требует специфических углов и не объясняет столь часто встречающееся явление. Короче, гипотеза о линейном frame-dragging имеет право быть. В теории её даже относительно легко можно проверить в солнечной системе, запустив интерферометр LISA по полярной орбите Солнца, чтобы одно плечо находилось над полюсом, а другое нет. Конечно, Солнце это даже близко не нейтронная звезда, и вращается оно крайне слоупочно, но какие-то микро искажения можно заметить.

А если оно так, то для сверхсветового полёта требуется воспроизвести этот поток пространства, как это делают быстровращающиеся высокогравитирующие объекты. Каким вентилятором это сделать - не имею ни малейшего понятия, но если нейтронные звёзды гравитационно взаимодействуют с жидким пространством - значит этот механизм на микроуровне можно расковырять и повторить.
483 669846
>>69822

> Есть серьёзные основания предполагать, что пространство-время это не тянущаяся резинка, а сорт оф сверхтекучая жидкость.


Это эфир?
484 669850
>>63052
Предлогаю построить коллайдер по земному экватору.
485 670035
>>69749
За щеку тебе высрал, пидорашка.
486 670044
Варп никогда не будет построен, как и чероточины. Смиритесь уже с законами физики. Скорость света это предел.
487 670051
Прочитал статью про систематическую ошибку выжившего
https://ru.wikipedia.org/wiki/Систематическая_ошибка_выжившего
и нихуя не понимаю.
Допустим у нас есть лабиринт с тысячами выходов и сотнями триллиардов тупиков.
Согласно выводам систематической ошибки выгоднее получить карту ведущую в один из тупиков, чем карту ведущую к одному из выходов? Или я что-то не так понял?
488 670055
>>70051
Никаких "выводов" у ошибки выжившего нет и быть не может, это просто феномен восприятия, который можно иметь в виду. Пример твой вообще никакой логики не имеет.
489 670061
>>70055
И в чем тогда смысл плодить сущности? Ошибку выжившего можно просто отнести к нерепрезентативной выборке и вообще никогда не брать в расчет.
490 670063
>>70061
Эта ошибка о причинах нерепрезентативной выборки как раз
491 670071
>>70063
Ну вообще-то есть критерии репрезентативной выборки.
492 670203
Пузыря Альбукерке не может быть.
Забудьте.
Пруфы: https://www.youtube.com/watch?v=cZF9LE48i14
493 670228
>>70203
Вот это лекция, очень интересно! Кто это запостил, тот напыщенный анон с пробелами нет, не пробелами - зияющими дырами в познаниях о Гномах Брауна?
494 670285
Как космонавты видят космос? Могут ли они, если отвернутся от Солнца, Земли и Луны, увидеть туманности, диск млечного пути? Ну и вообще, будь я в какой-то туманности и не смотря на яркие обьекты, видел бы космсос как на пикрл, или просто черную хуету с парой самых ярких звезд?

Много втыкал в спейс енджин, но никак не могу понять его настройки експозиции - ведь даже в ручном режиме он ее сам подкручивает так, что звезды хуй увидишь если в поле зрения будет поверхность планеты.

пиздец абу дегенерат, сначала аплоадит картинки весом в 10мб на сервер и только потом проверяет капчу. Так и задудосить дебила этого можно
495 670306
>>70203

>не может быть.



Всегда надо делать оговорку "при существующем уровне знаний"
496 670340
Какие уплотнители используют в космосе? Силикон как и на земле?
497 670356
>>70285
увидишь только млечный путь и ближайшие галактики, туманности едва ли. разве что что-то с самой большой поверхностной яркостью
498 670360
>>70285

>Как космонавты видят космос?


Примерно так же, как ты, если уедешь куда-нибудь в поля или тайгу ночью подальше от населенных пунктов и дорог, а лучше в горы.

>видел бы космсос как на пикрл


нет. эти картинки не только получены с нереальными экспозициями, еще и дополнительно обработаны, с усилением цветов, в ряде случаем еще и с наложениями невидимых диапазонов волн в искусственных цветах. То есть все это реально существует, но недоступно наблюдению в таком виде человеческим глазом.

>абу дегенерат


жи есть. эта капча с отгадывание циферок из которых половину нихуя не видно уже просто доебала
499 670366
>>70360
Еще нельзя забывать о зуме. Основная масса этих знаменитых картинок на реальном небе не занимают больше каких-нибудь угловых минут, это порядок размеров примерно в 1/10 размеров луны на небе, для человеческого глаза просто пятнышко.
500 670388
>>70360

>Примерно так же, как ты, если уедешь куда-нибудь в поля или тайгу ночью подальше от населенных пунктов и дорог, а лучше в горы.


Ну я был в Карпатах, в сравнении с городом просто немного больше звезд, но такого как на пик1 и близко не видел. И чет подозреваю, что даже нашу и другие галактики увидеть можно только через фотоаппарат.

>нет. эти картинки не только получены с нереальными экспозициями, еще и дополнительно обработаны, с усилением цветов, в ряде случаем еще и с наложениями невидимых диапазонов волн в искусственных цветах. То есть все это реально существует, но недоступно наблюдению в таком виде человеческим глазом.


Немножко напрягает это. Хочу видеть космос не таким каким он может быть на фото, а таким, каким я бы его увидел своими глазами.
Ну и раз в реальности я не могу видеть диск нашей галактики, допустим, я бы стоял относительно галактики в том же месте, что и на пик2. Увидел бы я тогда примерно такую же картинку? Увидел бы я другие, более далекие галактики?

Алсо, мб оффтоп вопрос, но можно ли настроить спейненджин чтобы экспозиция в нем была как в человеческих глазах?
70b77288afec7c7c406e4d509c0bdd27[2].jpg211 Кб, 1100x731
501 670396
>>70388
Потому что на твоей картинке опять же очень завышенная в сравнении с человеческим глазом экспозиция, достигнутая, правда, судя по отсутствию мазни на земле, комбинацией умеренной выдержки и очень хорошего ISO. Человек бы тут увидел просто черное небо с белесой полосой и звездами, да слабое зарево на горизонте. Рандомный чел, увидев млечный путь, вообще примет его за облака, так что ничего удивительного в твоих историях.

>Немножко напрягает это. Хочу видеть космос не таким каким он может быть на фото


Но зачем? Он был бы точно таким же в большинстве фоток, но гораздо более унылым. Либо ты бы вообще не увидел ничего (для тех случаев, где визуализируются гамма/УФ/ИК наблюдения)

>Увидел бы я тогда примерно такую же картинку


Нет. Открою тебе секрет - ты именно "в том самом месте", по отношению к галактике Андромеды. На небе она сравнительно огромна - целых 6 раз больше луны. Но ее невозможно видеть так как ты хочешь, потому что для глаза она тусклая.
За спейэнжин не ебу. Наверное можно, ведь это программа. Только скорее всего нахуй не нужно.
502 670515
Поясните насчёт капельных излучателей.

1. Капли же улетают в сборник под действием ускорения, создаваемого ДУ? Т.е. реактор может работать только на движение аппарата, а не запитывать какую-нибудь энергоёмкую ПН? Или они под давлением метко струят в каплесборник?

2. А как же снос капель заряженными частицами и возмущениями, в общем вот то что останавливало запил капельников ещё в восьмидесятых, когда концепция появилась.

3. Какую такую магию они юзают, чтобы жидкость не испарялась нахуй в вакууме сразу же?
503 670539
>>670538 (OP)
В тред тупых вопросов, дебил. >>62981 (OP)
504 670616
>>70515
1. Нет. Электростатической силой.
2. Магнитный контур + оптимальная электроемкость испарителя-поглотителя удерживает все говно.
3. Использовать жидкости с низким давлений паров: Ртуть, всякие эвтектики, силиконовые масла, фторорганика.
505 670623
>>70515
>>70616

> жидкости с низким давлений паров


> ртуть, фторорганика


лол. самое низкое давление паров (из металлов) при температуре плавления у олова, у ртути самое высокое, а фторорганика летает как угорелая даже при ОЧЕНЬ большой молекулярной масса. это троллинг такой был?
506 670627
>>70623
У ртути все равно давление паров ниже чем у масел или прочих жидкостей.
Впрочем оптимально подходит эвтектика различных щелочных металлов.
Так использовать надо не всякие простенькие фтороуглероды, а высокомолекулярные соединения.
507 670630
>>70627
так а почему ты назвал ртуть, а не олово, у которого еще и излучательная способность лучше, т.к. оно темнее? щелочные металлы тоже более летучие, а температура плавления выше
508 670631
>>70627
а вообще нафиг металлы, когда битум есть, он черный и будет светиться нормально в ИК
509 670633
>>70623

>а фторорганика летает как угорелая даже при ОЧЕНЬ большой молекулярной масса


Вообще-то лучшие вакуумные смазки как раз перфторполиэфирные, они вообще практически не испаряются и работают в очень широком диапазоне.
510 670635
>>70630
Рабочая температура все равно в районе 20-200 градусов по Цельсию.
На большие температуры уже не целесообразно, ибо уже обычный радиатор норм идет.
511 670639
>>70623

>фторорганика летает как угорелая даже при ОЧЕНЬ большой молекулярной масса


Чочо, дегазованные фторопласты, ПТФЭ, хлорированные арамиды повсеместно в космосе юзают
512 670640
>>70639
я почему-то про фреоны вспомнил раньше чем про фторопласт. его же вроде даже на 400 градусов нагревать нельзя?
513 670659
Я даже не знаю где спрашивать, поэтому напишу сюда. Года 2 назад зимой видел с приятелем на абсолютно чистом небе 2 постоянно сияющих обьекта, которые исходя из моих условных расчетов на такой высоте двигались очень и очень быстро, при этом мгновенно останавливаясь, меняли свою траекторию, как бы летая "дуэтом". Начал их снимать, в итоге изза темноты и плохого качества камеры были запечатлены только наши эмоции, в итоге после того как я начал снимать потанцевав вдвоём улетели так же быстро, как и прилетели. Ничего подобного никогда не видел, исключил все возможные варианты искусственного и нет происхождения этого представления, кроме шаровой молнии. Не может обьект, обладающий массой, лететь на такой высоте с такой скоростью и при этом не имея тормозного пути, останавливаться мгновенно, и так же мгновенно менять свою траекторию. Они даже не набирали скорость. Они летели тупо с одной, огромной скоростью, я думаю в разы быстрее спутника. Светились чуть ярче звезд в небе. Друг сказал что такое тоже видит в первые. Кто может предложить варианты, что это могло быть?
515 670756
>>70659
НЛО обычные. Видел такое над самарской лукой лет 10 назад как по небу гонялись красная и бело-зелёная точки. Помню что было весеннее равноденствие, но было ещё холодно, снежно и лёд стоял на Волге, поэтому мы с друзьями пошли вечером гулять по набережной.
516 670784
А чем парировали гипотезу, что бкп юпитера - это след ёбнувшего метеорита большого? типа рана кровоточащая, поэтично жеж
517 670800
>>70784
впервые слышу, интересно. но это что-то огромное должно было быть, следы от кометы шумейкера-леви прошли очень быстро
п.png27 Кб, 1176x690
518 670816
Может ли жирная на материю блуждающая планета быть пойманной какой-нибудь звездой так, чтобы своей изначальной массой (я про планету) она бы своей траекторией 3 начала падать на звезду, коснулась бы её - жирно потёршись об атмосферу звезды, коснувшись бочком поверхности или вообще нырнув на пару секунд под поверхность - и вылетела бы, но уже с потерей инерции из-за аэроторможения и очевидного сброса материи из-за слишком близкого нахождения со звездой (траектория 2) таким образом, чтобы в итоге, уже изрядно растерявшая и в материи, и в инерции планетка осталась бы стабильно вращаться на орбите этой самой звезды (траектория 3)?
519 670822
>>70816
ну в теории может, но потеряв большую часть массы и оставив кучу мусора
520 670857
>>70816
Если звезда двойная, вполне возможно прохождение планеты сквозь перетекающее с одного компонента на другой вещество, с последующей стабилизацией орбиты вокруг центра масс.
521 670931
>>64931

> Почему на всех фотачках галактик по центру настолько охуенно светло, что там будто бы одна огромная звезда?


> Это там просто самое плотное скопление звёзд?


звезд в галактике везде примерно одинаково, просто там более молодые звезды и светят они ярче
522 670939
>>70931
>>64931
не слушай его, в гало больше звезд чем по краям в разы
523 671029
>>70800
ну например, пиздень, которая нам луну сотворила. её же нет теперь. всё сходится!
524 671046
>>70756
Я тоже в Поволжье живу. И тоже был снег и холодно. Не помню даты, но могу сказать что один из объектов мне показался красноватого цвета, как ты и написал
525 671214
Почему МКС не разрывает изнутри, если снаружи нет давления? Там же ебать должно быть металла толщиной с палец у корпуса.
526 671324
>>71214
у тебя камера в колесе в самом дешевом автомобиле держит такую же разницу давлений
527 671395
>>71046
Самарская область вообще на всякие аномалии богата. Но это уже совсем другая история и для другого раздела...
528 671800
Как перекачивают топливо в двигатель в невесомости?
529 671802
>>71800
пукают газиком, он куда надо выдавливает
530 671858
>>71802
То есть газом сдувают жирные капли жидкого кислорода или водорода/метана к отверстию из которого его заберет насос?
531 671859
>>71800

>Как перекачивают топливо в двигатель в невесомости?



Маленький движок ullage engines or ullage rockets
532 671863
>>71859
Топливом для них служат газы?
Вообще сколько ускорения и на какое время нужно создать чтобы криогенное топливо прилипло к дну бака?
Фэлкон9 делает так же? Я не заметил работу этих двигателей во время отделения первой ступени.
533 671924
>>71863
первая ступень закручивается вокруг своей оси + топливо не мгновенно приходит в хаотичное болтание по баку, наверное этого хватает для сохранения доступа
534 671925
>>71924
>>71863
вокруг поперечной оси
фикс
535 672181
>>71863

>Фэлкон9 делает так же?



Гуглим ullage engines falcon-9

Первая же ссылка - Cold nitrogen gas ullage motors. Если не понимаем эльфийский - гугл транслэйт умеет переводить целые страницы.
536 672241
Человек летает в космос уже шестьдесят лет. Чем эти полёты с людьми помогли науке? Что с их помощью было открыто? Какие результаты короч.
537 672317
>>72241
Узнали что человек может покинуть атмосферу и вернуться почти невредимым.
538 672643
539 676146
>>72317
Охуеть цирк
Тред утонул или удален.
Это копия, сохраненная 18 января 2022 года.

Скачать тред: только с превью, с превью и прикрепленными файлами.
Второй вариант может долго скачиваться. Файлы будут только в живых или недавно утонувших тредах. Подробнее

Если вам полезен архив М.Двача, пожертвуйте на оплату сервера.
« /spc/В начало тредаВеб-версияНастройки
/a//b//mu//s//vg/Все доски